You are on page 1of 68

3920_Ch01_002-026 23/01/14 9:20 AM Page 22

22 Part One | Background

Study Questions
1. In the urinalysis laboratory the primary source in the 8. An acceptable disinfectant for blood and body fluid
chain of infection would be: decontamination is:
A. Patients A. Sodium hydroxide
B. Needlesticks B. Antimicrobial soap
C. Specimens C. Hydrogen peroxide
D. Biohazardous waste D. Sodium hypochlorite

2. The best way to break the chain of infection is: 9. Proper handwashing includes all of the following
except:
A. Hand sanitizing
A. Using warm water
B. Personal protective equipment
B. Rubbing to create a lather
C. Aerosol prevention
C. Rinsing hands in a downward position
D. Decontamination
D. Turning on the water with a paper towel
3. The current routine infection control policy developed
10. Centrifuging an uncapped specimen may produce a
by CDC and followed in all health-care settings is:
biologic hazard in the form of:
A. Universal Precautions
A. Vectors
B. Isolation Precautions
B. Sharps contamination
C. Blood and Body Fluid Precautions
C. Aerosols
D. Standard Precautions D. Specimen contamination
4. An employee who is accidentally exposed to a possible 11. An employee who accidentally spills acid on his arm
blood-borne pathogen should immediately: should immediately:
A. Report to a supervisor A. Neutralize the acid with a base
B. Flush the area with water B. Hold the arm under running water for 15 minutes
C. Clean the area with disinfectant C. Consult the MSDSs
D. Receive HIV prophylaxis D. Wrap the arm in gauze and go to the emergency
5. Personnel in the urinalysis laboratory should wear lab department
coats that: 12. When combining acid and water, ensure that:
A. Do not have buttons A. Acid is added to water
B. Are fluid-resistant B. Water is added to acid
C. Have short sleeves C. They are added simultaneously
D. Have full-length zippers D. Water is slowly added to acid
6. All of the following should be discarded in biohazardous 13. An employee can learn the carcinogenic potential of
waste containers except: potassium chloride by consulting the:
A. Urine specimen containers A. Chemical hygiene plan
B. Towels used for decontamination B. Material safety data sheets
C. Disposable lab coats C. OSHA standards
D. Blood collection tubes D. Urinalysis procedure manual

7. An employer who fails to provide sufficient gloves for 14. Employees should not work with radioisotopes if
the employees may be fined by the: they are:
A. CDC A. Wearing contact lenses
B. NFPA B. Allergic to iodine
C. Sensitive to latex
C. OSHA
D. Pregnant
D. FDA
3920_Ch01_002-026 23/01/14 9:20 AM Page 23

Chapter 1 | Safety and Quality Assessment 23

15. All of the following are safe to do when removing the 23. The classification of a fire that can be extinguished with
source of an electric shock except: water is:
A. Pulling the person away from the instrument A. Class A
B. Turning off the circuit breaker B. Class B
C. Using a glass container to move the instrument C. Class C
D. Unplugging the instrument D. Class D
16. The acronym PASS refers to: 24. Employers are required to provide free immunization for:
A. Presence of vital chemicals A. HIV
B. Operation of a fire extinguisher B. HTLV-1
C. Labeling of hazardous material C. HBV
D. Presence of radioactive substances D. HCV
17. The system used by firefighters when a fire occurs in the 25. A possible physical hazard in the hospital is:
laboratory is: A. Wearing closed-toed shoes
A. MSDS B. Not wearing jewelry
B. RACE C. Having short hair
C. NFPA D. Running to answer the telephone
D. PASS
26. Quality assessment refers to:
18. A class ABC fire extinguisher contains: A. Analysis of testing controls
A. Sand B. Increased productivity
B. Water C. Precise control results
C. Dry chemicals D. Quality of specimens and patient care
D. Acid
27. During laboratory accreditation inspections, procedure
19. The first thing to do when a fire is discovered is to: manuals are examined for the presence of:
A. Rescue persons in danger A. Critical values
B. Activate the alarm system B. Procedure references
C. Close doors to other areas C. Procedures for specimen preservation
D. Extinguish the fire if possible D. All of the above
20. If a red rash is observed after removing gloves, the 28. As supervisor of the urinalysis laboratory, you have just
employee: adopted a new procedure. You should:
A. May be washing her hands too often A. Put the package insert in the procedure manual
B. May have developed a latex allergy B. Put a complete, referenced procedure in the manual
C. Should apply cortisone cream C. Notify the microbiology department
D. Should not rub the hands so vigorously D. Put a cost analysis study in the procedure manual
21. Pipetting by mouth is: 29. Indicate whether each of the following would be consid-
A. Acceptable for urine but not serum ered a 1) preexamination, 2) examination, or 3) postex-
amination factor by placing the appropriate number in
B. Not acceptable without proper training
the blank:
C. Acceptable for reagents but not specimens
_____ Reagent expiration date
D. Not acceptable in the laboratory
_____ Rejecting a contaminated specimen
22. The NPFA classification symbol contains information on _____ Constructing a Levy-Jennings chart
all of the following except:
_____ Telephoning a positive Clinitest result on a
A. Fire hazards newborn
B. Biohazards _____ Calibrating the centrifuge
C. Reactivity _____ Collecting a timed urine specimen
D. Health hazards
3920_Ch01_002-026 23/01/14 9:20 AM Page 24

24 Part One | Background

30. The testing of sample from an outside agency and the com- 33. When a new bottle of QC material is opened, what
parison of results with participating laboratories is called: information is placed on the label?
A. External QC A. The supervisor’s initials
B. Electronic QC B. The lot number
C. Internal QC C. The date and the laboratory worker’s initials
D. Proficiency testing D. The time the bottle was opened
31. A color change that indicates that a sufficient amount of 34. When a control is run, what information is
patient’s specimen or reagent is added correctly to the documented?
test system would be an example of: A. The lot number
A. External QC B. Expiration date of the control
B. Equivalent QC C. The test results
C. Internal QC D. All of the above
D. Proficiency testing
32. What steps are taken when the results of reagent strip
QC are outside of the stated confidence limits?
A. Check the expiration date of the reagent strip
B. Run a new control
C. Open a new reagent strips container
D. All of the above

Case Studies and Clinical Situations


1. State a possible reason for an accreditation team to 3. The medical technologist was assigned to test 10 urine
report a deficiency in the following situations: specimens chemically. She removed 10 strips from the
container and proceeded with testing. Several patients’
a. The urine microscopic reporting procedure has
urine indicated a trace positive glucose in the urine.
been recently revised.
She then opened a new bottle of reagent dipsticks and
b. An unusually high number of urine specimens are proceeded to perform the QC. The negative control
being rejected because of improper collection. also tested as a trace positive for glucose. The medical
c. A key statement is missing from the Clinitest technologist consulted the supervisor. The supervisor
procedure. ran the QC and the results were in the correct range.
After observing the medical technologist’s technique,
d. Open control bottles in the refrigerator are examined. the supervisor realized that the medical technologist
2. As the new supervisor of the urinalysis section, you was waiting too long to read the glucose results and
encounter the following situations. Explain whether therefore reporting erroneous results.
you would accept them or take corrective action. a. What is wrong with this scenario?
a. You are told that the supervisor always performs b. Who should run the QC for each test? Why?
the CAP proficiency survey. c. When should controls be run?
b. QC is not performed daily on the reagent strips. d. What do you do when the QC is out of range?
c. The urinalysis section is primarily staffed by e. When can you report patient results?
personnel assigned to other departments for whom
you have no personnel data.
3920_Ch01_002-026 23/01/14 9:20 AM Page 25

Chapter 1 | Safety and Quality Assessment 25

4. An outpatient urine specimen was delivered to the b. Where would the information concerning what
laboratory at 0800 and placed on the counter in the should have been done with this specimen be
Urinalysis department. The medical laboratory scientist found and the criteria for rejection?
performed urinalysis on the specimen at 1130. The c. What QA procedure may have detected this error?
following results were abnormal:
d. What form will need to be completed for this
Clarity: Cloudy scenario?
pH: 9.0 e. How might this affect this patient’s care?
Nitrite: Positive f. How will the corrected results be documented?
The patient was a known diabetic; however, the
glucose result was negative.
a. What could be a possible cause for the abnormal
results?
3920_Ch02_027-038 23/01/14 10:08 AM Page 36

36 Part One | Background

PROCEDURE 2-4
Urine Drug Specimen Collection Procedure records the in-range temperature on the COC form
1. The collector washes hands and wears gloves. (COC step 2). If the specimen temperature is out of
range or the specimen is suspected of having been
2. The collector adds bluing agent (dye) to the toilet
diluted or adulterated, a new specimen must be
water reservoir to prevent an adulterated specimen.
collected and a supervisor notified.
3. The collector eliminates any source of water other
12. The specimen must remain in the sight of the donor and
than toilet by taping the toilet lid and faucet handles.
collector at all times.
4. The donor provides photo identification or positive
13. With the donor watching, the collector peels off the
identification from employer representative.
specimen identification strips from the COC form (COC
5. The collector completes step 1 of the chain-of-custody step 3) and puts them on the capped bottle, covering
(COC) form and has the donor sign the form. both sides of the cap.
6. The donor leaves his or her coat, briefcase, and/or 14. The donor initials the specimen bottle seals.
purse outside the collection area to avoid the possibil-
15. The date and time are written on the seals.
ity of concealed substances contaminating the urine.
16. The donor completes step 4 on the COC form.
7. The donor washes his or her hands and receives a
specimen cup. 17. The collector completes step 5 on the COC form.
8. The collector remains in the restroom but outside the 18. Each time the specimen is handled, transferred,
stall, listening for unauthorized water use, unless a or placed in storage, every individual must be
witnessed collection is requested. identified and the date and purpose of the change
recorded.
9. The donor hands specimen cup to the collector.
Transfer is documented. 19. The collector follows laboratory-specific instructions for
packaging the specimen bottles and laboratory copies of
10. The collector checks the urine for abnormal color and
the COC form.
for the required amount (30 to 45 mL).
20. The collector distributes the COC copies to appropriate
11. The collector checks that the temperature strip on the
personnel.
specimen cup reads 32.5°C to 37.7°C. The collector

Log on to 3. Torora, GJ, and Anagnostakos, NP: Principles of Anatomy


www.fadavis.com/strasinger and Physiology, ed 6, Harper & Row, New York, 1990,
for additional content related p. 51.
to this chapter. 4. Becton, Dickinson and Company: BD Vacutainer Urine Products
for collection, storage, and transport of urine specimens. Product
Circular, 2011.
References 5. Baer, DM: Glucose tolerance test: Tips from the clinical experts.
1. Herman, JR: Urology: A View Through the Retrospectroscope. Medical Laboratory Observer, Sept. 2003.
Harper & Row, Hagerstown, MD, 1973. 6. Rous, SN: The Prostate Book. Consumers Union, Mt. Vernon,
2. Clinical and Laboratory Standards Institute (formerly NCCLS), NY, 1988.
Approved Guideline GP16-A3: Urinalysis and Collection, 7. Stevermer, JJ, and Easley, SK: Treatment of prostatitis. Am Fam
Transportation, and Preservation of Urine Specimens; Approved Physician 61(10), 2000.
Guideline—ed 3, CLSI, Wayne, PA, 2009.

Study Questions
1. The average daily output of urine is: 2. An unidentified fluid is received in the laboratory with a
A. 200 mL request to determine whether the fluid is urine or another
body fluid. Using routine laboratory tests, what tests
B. 500 mL would determine that the fluid is most probably urine?
C. 1200 mL A. Glucose and ketones
D. 2500 mL B. Urea and creatinine
C. Uric acid and amino acids
D. Protein and amino acids
3920_Ch02_027-038 23/01/14 10:08 AM Page 37

Chapter 2 | Introduction to Urinalysis 37

3. The primary inorganic substance found in urine is: 11. The primary advantage of a first morning specimen over
A. Sodium a random specimen is that it:
B. Phosphate A. Is less contaminated
C. Chloride B. Is more concentrated
D. Calcium C. Is less concentrated
D. Has a higher volume
4. A patient presenting with polyuria, nocturia, polydipsia,
and a low urine specific gravity is exhibiting symptoms of: 12. If a routine urinalysis and a culture are requested on a
A. Diabetes insipidus catheterized specimen, then:
B. Diabetes mellitus A. Two separate containers must be collected
C. Urinary tract infection B. The routine urinalysis is performed first
D. Uremia C. The patient must be recatheterized
D. The culture is performed first
5. A patient with oliguria might progress to having:
13. If a patient fails to discard the first specimen when
A. Nocturia
collecting a timed specimen the:
B. Polyuria
A. Specimen must be recollected
C. Polydipsia
B. Results will be falsely elevated
D. Anuria
C. Results will be falsely decreased
6. All of the following are characteristics of recommended D. Both A and B
urine containers except:
14. The primary cause of unsatisfactory results in an un-
A. A flat bottom preserved routine specimen not tested for 8 hours is:
B. A capacity of 50 mL A. Bacterial growth
C. A snap-on lid B. Glycolysis
D. Are disposable C. Decreased pH
7. Labels for urine containers are: D. Chemical oxidation
A. Attached to the container 15. Prolonged exposure of a preserved urine specimen to
B. Attached to the lid light will cause:
C. Placed on the container prior to collection A. Decreased glucose
D. Not detachable B. Increased cells and casts
8. A urine specimen may be rejected by the laboratory for C. Decreased bilirubin
all of the following reasons except the fact that the: D. Increased bacteria
A. Requisition states the specimen is catheterized 16. Which of the following would be least affected in a
B. Specimen contains toilet paper specimen that has remained unpreserved at room
C. Label and requisition do not match temperature for more than 2 hours?
D. Outside of the container has fecal material A. Urobilinogen
contamination B. Ketones
C. Protein
9. A cloudy specimen received in the laboratory may have
been preserved using: D. Nitrite
A. Boric acid 17. Bacterial growth in an unpreserved specimen will:
B. Chloroform A. Decrease clarity
C. Refrigeration B. Increase bilirubin
D. Formalin C. Decrease pH
10. For general screening the most frequently collected D. Increase glucose
specimen is a: 18. The most sterile specimen collected is a:
A. Random one A. Catheterized
B. First morning B. Midstream clean-catch
C. Midstream clean-catch C. Three-glass
D. Timed D. Suprapubic aspiration
3920_Ch02_027-038 23/01/14 10:08 AM Page 38

38 Part One | Background

19. Which of the following would not be given to a patient 20. Urine specimen collection for drug testing requires the
prior to the collection of a midstream clean-catch specimen? collector to do all of the following except:
A. Sterile container A. Inspect the specimen color
B. Iodine cleanser B. Perform reagent strip testing
C. Antiseptic towelette C. Read the specimen temperature
D. Instructions D. Fill out a chain-of-custody form

Case Studies and Clinical Situations


1. A patient brings a first morning specimen to the 5. A worker suspects that he or she will be requested to
laboratory at 1:00 p.m. collect an unwitnessed urine specimen for drug analysis.
a. How could this affect the urinalysis results? He or she carries a substitute specimen in his or her
pocket for 2 days before being told to collect the speci-
b. What could the patient say that would make the men. Shortly after the worker delivers the specimen to
specimen satisfactory for testing? the collector, he or she is instructed to collect another
2. A patient collecting a midstream clean-catch specimen specimen.
voids immediately into the container. a. What test was performed on the specimen to
a. How could this affect the clarity of the specimen? determine possible specimen manipulation?
b. How could this affect the microscopic examination? b. How was the specimen in this situation affected?
c. If a specimen for drug analysis tests positive, state a
3. A patient brings a 24-hour timed specimen to the possible defense related to specimen collection and
laboratory and reports that he or she forgot to collect handling that an attorney might employ.
a specimen voided during the night.
d. How can this defense be avoided?
a. How will this affect the results of a quantitative test
for creatinine?
b. What should the patient be told to do?
4. You receive a urine preservative tube for culture
containing a volume of specimen that is considerably
below the minimum fill line.
a. Could this affect the culture?
b. Why?
3920_Ch03_039-057 23/01/14 10:10 AM Page 53

Chapter 3 | Renal Function 53

the ammonium concentration can be calculated as the differ-


HISTORICAL NOTE
ence between the titratable acidity and the total acidity.
Phenolsulfonphthalein Test
Log on to
www.fadavis.com/strasinger
Historically, excretion of the dye phenolsulfonphthalein for additional content related
(PSP) was used to evaluate these functions. Standardiza- to this chapter.
tion and interpretation of PSP results are difficult, how-
ever, because of interference by medications, elevated References
waste products in patients’ serum, the necessity to obtain 1. Berger, A: Renal function and how to assess it. Brit J Med
several very accurately timed urine specimens, and the 321:1444, 2000.
possibility of producing anaphylactic shock. Therefore, 2. Pincus, MR, Preuss, HG, and Henry, JB: Evaluation of renal func-
tion and water, electrolyte and acid-base balance. In Henry, JB
the PSP test is not currently performed. (ed): Clinical Diagnosis and Management by Laboratory Meth-
ods. WB Saunders, Philadelphia, 1996.
3. Levey, AS, et al: A new equation to estimate glomerular filtration
rate. Ann Intern Med 150(9):601–612, 2009.
The inability to produce an acid urine in the presence of 4. Laterza, OE, Price, CP, and Scott, MG: Cystatin C: An improved
metabolic acidosis is called renal tubular acidosis. This condi- estimator of glomerular filtration rate? Clin Chem 48(5):
tion may result from impaired tubular secretion of hydrogen 699–707, 2002.
ions associated with the proximal convoluted tubule or defects 5. Tan, GS, et al: Clinical usefulness of cystatin C for the estimation
in ammonia secretion associated with the distal convoluted of glomerular filtration rate in type 1 diabetes. Crit Care
9(2):139–143, 2005.
tubule. 6. Inker, LA: Estimating glomerular filtration rate from serum crea-
Urine pH, titratable acidity, and urinary ammonia meas- tinine and cystatin C. N Engl J Med 367:20–29, 2012.
urements can be used to determine the defective function. The 7. Foley, K: Beta 2 microglobulin: a facultative marker. Advance for
tests can be run simultaneously on either fresh or toluene- MLP, Sept 30, 2008, p 13.
preserved urine specimens collected at 2-hour intervals from 8. Chachati, A, et al: Rapid method for the measurement of differ-
ential renal function: Validation. J Nucl Med 28(5): 829–836,
patients who have been primed with an acid load consisting 1987.
of oral ammonium chloride. By titrating the amount of free H+ 9. Daves, BB, and Zenser, TV: Evaluation of renal concentrating and
(titratable acidity) and then the total acidity of the specimen, diluting ability. Clin Lab Med 13(1):131–134, 1993.

Study Questions
1. The type of nephron responsible for renal concentration 4. Filtration of protein is prevented in the glomerulus by:
is the: A. Hydrostatic pressure
A. Cortical B. Oncotic pressure
B. Juxtaglomerular C. Renin
2. The function of the peritubular capillaries is: D. The glomerular filtration barrier
A. Reabsorption 5. The renin-angiotensin-aldosterone system is responsible for
all of the following except:
B. Filtration
A. Vasoconstriction of the afferent arteriole
C. Secretion
B. Vasoconstriction of the efferent arteriole
D. Both A and C
C. Reabsorbing sodium
3. Blood flows through the nephron in the following order: D. Releasing aldosterone
A. Efferent arteriole, peritubular capillaries, vasa recta,
6. The primary chemical affected by the renin-angiotensin-
afferent arteriole
aldosterone system is:
B. Peritubular capillaries, afferent arteriole, vasa recta, A. Chloride
efferent arteriole
B. Sodium
C. Afferent arteriole, peritubular capillaries, vasa recta,
C. Potassium
efferent arteriole
D. Hydrogen
D. Efferent arteriole, vasa recta, peritubular capillaries,
afferent arteriole
3920_Ch03_039-057 23/01/14 10:10 AM Page 54

54 Part One | Background

7. Secretion of renin is stimulated by: 14. ADH regulates the final urine concentration by
A. Juxtaglomerular cells controlling:
B. Angiotensin I and II A. Active reabsorption of sodium
C. Macula densa cells B. Tubular permeability
D. Circulating angiotensin-converting enzyme C. Passive reabsorption of urea
D. Passive reabsorption of chloride
8. The hormone aldosterone is responsible for:
A. Hydrogen ion secretion 15. Decreased production of ADH:

B. Potassium secretion A. Produces a low urine volume


B. Produces a high urine volume
C. Chloride retention
C. Increases ammonia excretion
D. Sodium retention
D. Affects active transport of sodium
9. The fluid leaving the glomerulus has a specific
gravity of: 16. Bicarbonate ions filtered by the glomerulus are returned
to the blood:
A. 1.005
A. In the proximal convoluted tubule
B. 1.010
B. Combined with hydrogen ions
C. 1.015
C. By tubular secretion
D. 1.020
D. All of the above
10. For active transport to occur, a chemical:
17. If ammonia is not produced by the distal convoluted
A. Must combine with a carrier protein to create tubule, the urine pH will be:
electrochemical energy
A. Acidic
B. Must be filtered through the proximal convoluted
B. Basic
tubule
C. Must be in higher concentration in the filtrate than 18. Place the appropriate letter in front of the following clear-
in the blood ance substances:
D. Must be in higher concentration in the blood than in A. Exogenous
the filtrate B. Endogenous
11. Which of the tubules is impermeable to water? ____ beta2-microglobulin
A. Proximal convoluted tubule ____ creatinine
B. Descending loop of Henle ____ cystatin C
C. Ascending loop of Henle ____ 125I-iothalmate

D. Distal convoluted tubule 19. The largest source of error in creatinine clearance tests is:
A. Secretion of creatinine
12. Glucose will appear in the urine when the:
B. Improperly timed urine specimens
A. Blood level of glucose is 200 mg/dL
C. Refrigeration of the urine
B. Tm for glucose is reached
D. Time of collecting blood sample
C. Renal threshold for glucose is exceeded
D. All of the above 20. Given the following information, calculate the creatinine
clearance:
13. Concentration of the tubular filtrate by the countercur-
rent mechanism depends on all of the following 24-hour urine volume: 1000 mL; serum creatinine:
except: 2.0 mg/dL; urine creatinine: 200 mg/dL
A. High salt concentration in the medulla 21. Clearance tests used to determine the glomerular filtration
B. Water-impermeable walls of the ascending loop of rate must measure substances that are:
Henle A. Not filtered by the glomerulus
C. Reabsorption of sodium and chloride from the B. Completely reabsorbed by the proximal convoluted
ascending loop of Henle tubule
D. Reabsorption of water in the descending loop of C. Secreted in the distal convoluted tubule
Henle D. Neither reabsorbed or secreted by the tubules
3920_Ch03_039-057 23/01/14 10:10 AM Page 55

Chapter 3 | Renal Function 55

22. Performing a clearance test using radionucleotides: 29. After controlled fluid intake, the urine-to-serum osmolarity
A. Eliminates the need to collect urine ratio should be at least:
B. Does not require an infusion A. 1:1
C. Provides visualization of the filtration B. 2:1
D. Both A and C
C. 3:1
23. Variables that are included in the MDRD-IDSM estimated D. 4:1
creatinine clearance calculations include all of the
following except: 30. Calculate the free water clearance from the following
A. Serum creatinine results:
B. Weight urine volume in 6 hours: 720 mL; urine osmolarity:
C. Age 225 mOsm; plasma osmolarity: 300 mOsm
D. Gender
31. To provide an accurate measure of renal blood flow, a test
24. An advantage to using cystatin C to monitor GFR is that: substance should be completely:
A. It does not require urine collection
A. Filtered by the glomerulus
B. It is not secreted by the tubules
B. Reabsorbed by the tubules
C. It can be measured by immunoassay
C. Secreted when it reaches the distal convoluted
D. All of the above
tubule
25. Solute dissolved in solvent will:
D. Cleared on each contact with functional renal
A. Raise the vapor pressure
tissue
B. Lower the boiling point
C. Decrease the osmotic pressure 32. Given the following data, calculate the effective renal
D. Lower the freezing point plasma flow:

26. Substances that may interfere with freezing point meas- urine volume in 2 hours: 240 mL; urine PAH: 150 mg/dL;
urement of urine and serum osmolarity include all of the plasma PAH: 0.5 mg/dL
following except:
33. Renal tubular acidosis can be caused by the:
A. Ethanol
A. Production of excessively acidic urine due to
B. Lactic acid
increased filtration of hydrogen ions
C. Sodium
D. Lipids B. Production of excessively acidic urine due to
increased secretion of hydrogen ions
27. Clinical osmometers use NaCl as a reference solution
because: C. Inability to produce an acidic urine due to impaired
A. 1 g molecular weight of NaCl will lower the freezing production of ammonia
point 1.86oC D. Inability to produce an acidic urine due to increased
B. NaCl is readily frozen production of ammonia
C. NaCl is partially ionized similar to the composition 34. Tests performed to detect renal tubular acidosis after
of urine
administering an ammonium chloride load include all
D. 1 g equivalent weight of NaCl will raise the freezing
of the following except:
point 1.86oC
A. Urine ammonia
28. The normal serum osmolarity is:
B. Arterial pH
A. 50 to 100 mOsm
B. 275 to 300 mOsm C. Urine pH
C. 400 to 500 mOsm D. Titratable acidity
D. 3 times the urine osmolarity
3920_Ch03_039-057 23/01/14 10:10 AM Page 56

56 Part One | Background

Case Studies and Clinical Situations


1. A 44-year-old man diagnosed with acute tubular necrosis 4. A laboratory is obtaining erratic serum osmolarity results
has a blood urea nitrogen of 60 mg/dL and a blood on a patient who is being monitored at 6 a.m., 12 p.m.,
glucose level of 100 mg/dL. A 2+ urine glucose is also 6 p.m., and 12 a.m. Osmolarities are not performed on
reported. the night shift; therefore, the midnight specimen is run at
the same time as the 6 a.m. specimen.
a. State the renal threshold for glucose.
a. What two reasons could account for these
b. What is the significance of the positive urine
discrepancies?
glucose and normal blood glucose?
b. What substance is causing the erratic results?
2. A patient develops a sudden drop in blood pressure. c. If a friend were secretly bringing the patient a pint of
a. Diagram the reactions that take place to ensure whiskey every night, would this affect the
adequate blood pressure within the nephrons. results? Explain your answer.
b. How do these reactions increase blood volume? 5. Following overnight (6 p.m. to 8 a.m.) fluid deprivation,
c. When blood pressure returns to normal, how does the the urine-to-serum osmolarity ratio in a patient who is
kidney respond? exhibiting polyuria and polydipsia is 1:1. The ratio re-
mains the same when a second specimen is tested at
3. A physician would like to prescribe a nephrotoxic antibi- 10 a.m. ADH is then administered subcutaneously to
otic for a 60-year-old Caucasian man. The patient has a the patient, and the fluid deprivation is continued until
serum creatinine level of 1.5 mg/dL. 2 p.m., when another specimen is tested.
a. How can the physician determine whether it is safe to a. What disorder do these symptoms and initial laboratory
prescribe this medication before the patient leaves the results indicate?
office? b. If the urine-to-serum osmolarity ratio on the 2 p.m.
b. State two additional blood tests that the physician specimen is 3:1, what is the underlying cause of the
could use to continue monitoring this patient. patient’s disorder?
c. If the patient has a history of prostate malignancy, c. If the urine-to-serum osmolarity ratio on the 2 p.m.
would both of these methods provide reliable results? specimen remains 1:1, what is the underlying cause of
the patient’s disorder?
Explain your answer.
3920_Ch04_058-070 23/01/14 10:56 AM Page 67

Chapter 4 | Physical Examination of Urine 67

cause an unusual or pungent urine odor. Studies have shown References


that although everyone who eats asparagus produces an odor, 1. Henry, JB, Lauzon, RB, and Schumann, GB: Basic examination
only certain genetically predisposed people can smell the of urine. In Henry, JB (ed): Clinical Diagnosis and Management
odor.11 Common causes of urine odors are summarized in by Laboratory Methods. WB Saunders, Philadelphia, 1996.
Table 4–6. 2. Drabkin, DL: The normal pigment of urine: The relationship of
urinary pigment output to diet and metabolism. J Biol Chem
75:443–479, 1927.
3. Ostow, M, and Philo, S: The chief urinary pigment: The rela-
Table 4–6 Possible Causes of Urine Odor1
tionship between the rate of excretion of the yellow pigment
Odor Cause and the metabolic rate. Am J Med Sci 207:507–512, 1944.
4. Berman, L: When urine is red. JAMA 237:2753–2754, 1977.
Aromatic Normal 5. Reimann, HA: Re: Red urine. JAMA 241(22):2380, 1979.
6. Evans, B: The greening of urine: Still another “Cloret sign.”
Foul, ammonia-like Bacterial decomposition, urinary N Engl J Med 300(4):202, 1979.
tract infection 7. Bowling, P, Belliveau, RR, and Butler, TJ: Intravenous medica-
Fruity, sweet Ketones (diabetes mellitus, tions and green urine. JAMA 246(3):216, 1981.
8. Dealler, SF, et al: Purple urine bags. J Urol 142(3):769–770,
starvation, vomiting) 1989.
Maple syrup Maple syrup urine disease 9. Clinical and Laboratory Standards Institute (Formerly NCCLS)
Approved Guideline GP16-A3: Urinalysis and Collection,
Mousy Phenylketonuria Transportation, and Preservation of Urine Specimens;
Rancid Tyrosinemia Approved Guideline, ed 3, CLSI, Wayne, PA, 2009.
10. Smith, C, Arbogast, C, and Phillips, R: Effect of x-ray contrast
Sweaty feet Isovaleric acidemia media on results for relative density of urine. Clin Chem
Cabbage Methionine malabsorption 19(4):730–731, 1983.
11. Mitchell, SC, et al: Odorous urine following asparagus inges-
Bleach Contamination tion in man. Experimenta 43(4):382–383, 1987.

Log on to
www.fadavis.com/strasinger
for additional content related
to this chapter.

Study Questions
1. The concentration of a normal urine specimen can be 4. A urine specimen containing melanin will appear:
estimated by which of the following? A. Pale pink
A. Color B. Dark yellow
B. Clarity C. Blue-green
C. Foam D. Black
D. Odor 5. Specimens that contain hemoglobin can be visually dis-
tinguished from those that contain RBCs because:
2. The normal yellow color of urine is produced by:
A. Hemoglobin produces a clear, yellow specimen
A. Bilirubin
B. Hemoglobin produces a cloudy pink specimen
B. Hemoglobin
C. RBCs produce a cloudy red specimen
C. Urobilinogen D. RBCs produce a clear red specimen
D. Urochrome
6. A patient with a viscous orange specimen may have been:
3. The presence of bilirubin in a urine specimen produces a: A. Treated for a urinary tract infection
A. Yellow foam when shaken B. Taking vitamin B pills
B. White foam when shaken C. Eating fresh carrots
C. Cloudy specimen D. Taking antidepressants
D. Yellow-red specimen
3920_Ch04_058-070 23/01/14 10:56 AM Page 68

68 Part Two | Urinalysis

7. The presence of a pink precipitate in a refrigerated speci- 14. A cloudy urine specimen turns black upon standing and
men is caused by: has a specific gravity of 1.012. The major concern about
A. Hemoglobin this specimen would be:
B. Urobilin A. Color
C. Uroerythrin B. Turbidity
D. Beets C. Specific gravity
D. All of the above
8. Microscopic examination of a clear urine that produces a
white precipitate after refrigeration will show: 15. A specimen with a specific gravity of 1.035 would be con-
A. Amorphous urates sidered:
B. Porphyrins A. Isosthenuric
C. Amorphous phosphates B. Hyposthenuric
D. Yeast C. Hypersthenuric
D. Not urine
9. The color of urine containing porphyrins will be:
16. A specimen with a specific gravity of 1.001 would be con-
A. Yellow-brown
sidered:
B. Green
A. Hyposthenuric
C. Orange
B. Not urine
D. Port wine
C. Hypersthenuric
10. Which of the following specific gravities would be most D. Isosthenuric
likely to correlate with a pale yellow urine?
17. A strong odor of ammonia in a urine specimen could
A. 1.005
indicate:
B. 1.010
A. Ketones
C. 1.020
B. Normalcy
D. 1.030
C. Phenylketonuria
11. A urine specific gravity measured by refractometer is D. An old specimen
1.029, and the temperature of the urine is 14°C. The spe-
cific gravity should be reported as: 18. The microscopic of a clear red urine is reported as many
WBCs and epithelial cells. What does this suggest?
A. 1.023
A. Urinary tract infection
B. 1.027
B. Dilute random specimen
C. 1.029
C. Hematuria
D. 1.032
D. Possible mix-up of specimen and sediment
12. The principle of refractive index is to compare:
19. Which of the following would contribute the most to a
A. Light velocity in solutions with light velocity in urine osmolality?
solids
A. One osmole of glucose
B. Light velocity in air with light velocity in solutions
B. One osmole of urea
C. Light scattering by air with light scattering by
C. One osmole of sodium chloride
solutions
D. All contribute equally
D. Light scattering by particles in solution
20. Which of the following colligative properties is not stated
13. A correlation exists between a specific gravity by refrac-
correctly?
tometer of 1.050 and a:
A. The boiling pointing is raised by solute
A. 2+ glucose
B. The freezing point is raised by solute
B. 2+ protein
C. The vapor pressure is lowered by solute
C. First morning specimen
D. The osmotic pressure is raised by solute
D. Radiographic dye infusion
3920_Ch04_058-070 23/01/14 10:56 AM Page 69

Chapter 4 | Physical Examination of Urine 69

21. An osmole contains: 23. In the reagent strip specific gravity reaction the
A. One gram molecular weight of solute dissolved in polyelectrolyte:
one liter of solvent A. Combines with hydrogen ions in response to ion
B. One gram molecular weight of solute dissolved concentration
in one kilogram of solvent B. Releases hydrogen ions in response to ion
C. Two gram molecular weights of solute dissolved in concentration
one liter of solvent C. Releases hydrogen ions in response to pH
D. Two gram molecular weights of solute dissolved D. Combines with sodium ions in response to pH
in one kilogram of solvent
24. Which of the following will react in the reagent strip
22. The unit of osmolality measured in the clinical laboratory specific gravity test?
is the: A. Glucose
A. Osmole B. Radiographic dye
B. Milliosmole C. Protein
C. Molecular weight D. Chloride
D. Ionic charge

Case Studies and Clinical Situations


1. Given the following physical urinalysis results, determine c. If the specific gravity was also checked using osmome-
additional urinalysis results that may be abnormal. try, should the result agree with the laboratory or the
a. A green specimen with a strong foul odor of ammonia urology clinic results? Why or why not?
b. A pale yellow urine with a specific gravity of 1.030 3. State two pathologic causes of a clear red urine.
c. A dark yellow specimen with yellow foam a. State a method that could distinguish between the two
d. A cloudy red urine causes that does not require laboratory testing.

2. The urology clinic questions a urinalysis report from the 4. Mrs. Smith frequently shops at the farmer’s market near
laboratory. her home. She notices her urine has a red color and
brings a sample to her physician. The specimen tests
The laboratory report states that a reagent strip reading
negative for blood.
of a specific gravity of 1.020, protein 3 g/dL, and glucose
2 g/dL. The specific gravity in the urology clinic was a. What is a probable cause of Mrs. Smith’s red urine?
greater than 1.035. b. Mrs. Smith collects a specimen at the physician’s
a. Correct the refractometer reading to account for the office. The color is yellow and the pH is 5.5. Is
protein and glucose concentrations. What is the cor- this consistent with the previous answer? Why or
rected specific gravity? why not?
b. Do the specific gravities correlate? 5. Is a clear urine always normal? Explain your answer.
3920_Ch05_071-098 23/01/14 10:16 AM Page 92

92 Part Two | Urinalysis

Reaction Interference Log on to


www.fadavis.com/strasinger
The reagent strip specific gravity measures only ionic solutes, for additional content related
thereby eliminating the interference by the large organic mol- to this chapter.
ecules, such as urea and glucose, and by radiographic contrast
media and plasma expanders that are included in physical
measurements of specific gravity. This difference must be con-
References
sidered when comparing specific gravity results obtained by 1. Chemstrip 10UA product Insert, Roche Diagnostics, Indianapolis,
IN, 2004.
a different method. Elevated concentrations of protein slightly 2. Multistix Pro Reagent Strips Product Insert. Siemens Diagnos-
increase the readings as a result of protein anions. tics, Tarrytown, NY 2005.
Specimens with a pH of 6.5 or higher have decreased 3. TechniTips, Miles Diagnostics, Elkhart, IN, October, 1992.
readings caused by interference with the bromthymol blue in- 4. Clinical and Laboratory Standards Institute Approved Guide-
dicator (the blue-green readings associated with an alkaline line GP16-A3: Urinalysis and Collection, Transportation, and
Preservation of Urine Specimens; Approved Guideline, ed 3,
pH correspond to a low specific gravity reading). Therefore, CLSI, Wayne, PA, 2009.
manufacturers recommend adding 0.005 to specific gravity 5. College of American Pathologists, CAP Today, Confirmatory
readings when the pH is 6.5 or higher. The correction is per- testing. Chicago, IL. December 2011.
formed by automated strip readers. 6. Tips from the Clinical Experts, Medical Laboratory Observer.
Web site: http://www.mlo-online.com/articles/mlo0802tips.htm
7. Pugia, MJ, and Lott, JA: New developments in urinalysis strip
tests for proteins. In Bayer Encyclopedia of Urinalysis. Bayer
SUMMARY 5-22 Urine Specific Gravity Diagnostics, Elkhart, IN, 2002.
8. Bhuwnesh, A, et al: Microalbumin screening by reagent strip
Reagent Strip predicts cardiovascular risk in hypertension. J Hypertens 14:
223–228, 1992.
Reagents Multistix: Poly (methyl vinyl ether/maleic 9. Bianchi, S, et al: Microalbuminurea in essential hypertension.
anhydride) bromthymol blue J Nephrol 10(4):216–219, 1997.
10. Clinitek Microalbumin Reagent Strip Product Insert. Bayer
Chemstrip: Ethylene glycol diaminoethyl Diagnostics, Elkhart, IN, 2006.
ether tetraacetic acid, bromthymol 11. Benedict, SR: A reagent for the detection of reducing sugars.
blue J Biol Chem 5:485–487, 1909.
Sensitivity 1.000 to 1.030 12. Lane R, and Phillips, M: Rhabdomyolysis has many causes
including statins and may be fatal. Brit J Med 327:115–116,
Interference False-positive: High concentrations of 2003.
protein 13. Hager, CB, and Free, AH: Urine urobilinogen as a component
of routine urinalysis. Am J Med Technol 36(5):227–233, 1970.
False-negative: Highly alkaline urines 14. Wise, KA, Sagert, LA, and Grammens, GL: Urine leukocyte es-
(greater than 6.5) terase and nitrite tests as an aid to predict urine culture results.
Lab Med 15(3):186–187, 1984.

Study Questions
1. Leaving excess urine on the reagent strip after removing 3. Testing a refrigerated specimen that has not warmed to
it from the specimen will: room temperature will adversely affect:
A. Cause run-over between reagent pads A. Enzymatic reactions
B. Alter the color of the specimen B. Dye-binding reactions
C. Cause reagents to leach from the pads C. The sodium nitroprusside reaction
D. Not affect the chemical reactions D. Diazo reactions
2. Failure to mix a specimen before inserting the reagent 4. The reagent strip reaction that requires the longest reac-
strip will primarily affect the: tion time is the:
A. Glucose reading A. Bilirubin
B. Blood reading B. pH
C. Leukocyte reading C. Leukocyte esterase
D. Both B and C D. Glucose
3920_Ch05_071-098 23/01/14 10:16 AM Page 93

Chapter 5 | Chemical Examination of Urine 93

5. Quality control of reagent strips is performed: 12. All of the following will cause false-positive protein
A. Using positive and negative controls reagent strip values except:
B. When results are questionable A. Microalbuminuria
C. At least once every 24 hours B. Highly buffered alkaline urines
D. All of the above C. Delay in removing the reagent strip from the specimen
D. Contamination by quaternary ammonium compounds
6. All of the following are important to protect the integrity
of reagent strips except: 13. A patient with a 2+ protein reading in the afternoon is asked
to submit a first morning specimen. The second specimen
A. Removing the desiccant from the bottle
has a negative protein reading. This patient is:
B. Storing in an opaque bottle
A. Positive for orthostatic proteinuria
C. Storing at room temperature
B. Negative for orthostatic proteinuria
D. Resealing the bottle after removing a strip
C. Positive for Bence Jones protein
7. The principle of the reagent strip test for pH is the: D. Negative for clinical proteinuria
A. Protein error of indicators 14. Testing for microalbuminuria is valuable for early detec-
B. Greiss reaction tion of kidney disease and monitoring patients with:
C. Dissociation of a polyelectrolyte A. Hypertension
D. Double indicator reaction B. Diabetes mellitus
8. A urine specimen with a pH of 9.0: C. Cardiovascular disease risk
A. Indicates metabolic acidosis D. All of the above
B. Should be recollected 15. The primary chemical on the reagent strip in the Micral-
C. May contain calcium oxalate crystals Test for microalbumin binds to:
D. Is seen after drinking cranberry juice A. Protein
B. Antihuman albumin antibody
9. In the laboratory, a primary consideration associated
with pH is: C. Conjugated enzyme
D. Galactoside
A. Identifying urinary crystals
B. Monitoring vegetarian diets 16. All of the following are true for the ImmunoDip test for
microalbumin except:
C. Determining specimen acceptability
A. Unbound antibody migrates farther than bound
D. Both A and C
antibody
10. Indicate the source of the following proteinurias by plac- B. Blue latex particles are coated with antihuman
ing a 1 for prerenal, 2 for renal, or 3 for postrenal in front albumin antibody
of the condition. C. Bound antibody migrates further than unbound
A. ____Microalbuminuria antibody
B. ____Acute phase reactants D. It utilizes an immunochromographic principle
C. ____Pre-eclampsia 17. The principle of the protein-high pad on the Multistix
D. ____Vaginal inflammation Pro reagent strip is the:
E. ____Multiple myeloma A. Diazo reaction
F. ____Orthostatic proteinuria B. Enzymatic dye-binding reaction
G. ____Prostatitis C. Protein error of indicators
11. The principle of the protein error of indicators reaction D. Microalbumin-Micral-Test
is that: 18. Which of the following is not tested on the Multistix Pro
A. Protein keeps the pH of the urine constant reagent strip?
B. Albumin accepts hydrogen ions from the indicator A. Urobilinogen
C. The indicator accepts hydrogen ions from B. Specific gravity
albumin C. Creatinine
D. Albumin changes the pH of the urine D. Protein-high
3920_Ch05_071-098 23/01/14 10:16 AM Page 94

94 Part Two | Urinalysis

19. The principle of the protein-low reagent pad on the Mul- 27. The most significant reagent strip test that is associated
tistix Pro is the: with a positive ketone result is:
A. Binding of albumin to sulphonphthalein dye A. Glucose
B. Immunologic binding of albumin to antibody B. Protein
C. Reverse protein error of indicators reaction C. pH
D. Enzymatic reaction between albumin and dye D. Specific gravity
20. The principle of the creatinine reagent pad on microal- 28. The primary reagent in the reagent strip test for ketones is:
bumin reagent strips is the: A. Glycine
A. Double indicator reaction B. Lactose
B. Diazo reaction C. Sodium hydroxide
C. Pseudoperoxidase reaction D. Sodium nitroprusside
D. Reduction of a chromogen
29. Ketonuria may be caused by all of the following except:
21. The purpose of performing an albumin:creatinine ratio A. Bacterial infections
is to:
B. Diabetic acidosis
A. Estimate the glomerular filtration rate
C. Starvation
B. Correct for hydration in random specimens
D. Vomiting
C. Avoid interference for alkaline urines
D. Correct for abnormally colored urines 30. Urinalysis on a patient with severe back and abdominal
pain is frequently performed to check for:
22. A patient with a normal blood glucose and a positive
A. Glucosuria
urine glucose should be further checked for:
B. Proteinuria
A. Diabetes mellitus
C. Hematuria
B. Renal disease
D. Hemoglobinuria
C. Gestational diabetes
D. Pancreatitis 31. Place the appropriate number or numbers in front of
each of the following statements. Use both numbers for
23. The principle of the reagent strip tests for glucose is the: an answer if needed.
A. Peroxidase activity of glucose 1. Hemoglobinuria
B. Glucose oxidase reaction 2. Myoglobinuria
C. Double sequential enzyme reaction A. ____ Associated with transfusion reactions
D. Dye-binding of glucose and chromogen B. ____ Clear red urine and pale yellow plasma
24. All of the following may produce false-negative glucose C. ____ Clear red urine and red plasma
reactions except: D. ____ Associated with rhabdomyolysis
A. Detergent contamination E. ____ Produces hemosiderin granules in urinary
B. Ascorbic acid sediments
C. Unpreserved specimens F. ____Associated with acute renal failure
D. Low urine temperature 32. The principle of the reagent strip test for blood is based
25. The primary reason for performing a Clinitest is to: on the:
A. Check for high ascorbic acid levels A. Binding of heme and a chromogenic dye
B. Confirm a positive reagent strip glucose B. Peroxidase activity of heme
C. Check for newborn galactosuria C. Reaction of peroxide and chromogen
D. Confirm a negative glucose reading D. Diazo activity of heme
26. The three intermediate products of fat metabolism in- 33. A speckled pattern on the blood pad of the reagent strip
clude all of the following except: indicates:
A. Acetoacetic acid A. Hematuria
B. Ketoacetic acid B. Hemoglobinuria
C. β -hydroxybutyric acid C. Myoglobinuria
D. Acetone D. All of the above
3920_Ch05_071-098 23/01/14 10:16 AM Page 95

Chapter 5 | Chemical Examination of Urine 95

34. List the following products of hemoglobin degradation 42. All of the following can cause a negative nitrite reading
in the correct order by placing numbers 1 to 4 in the except:
blank. A. Gram-positive bacteria
A. ____Conjugated bilirubin B. Gram-negative bacteria
B. ____Urobilinogen and stercobilinogen C. Random urine specimens
C. ____Urobilin D. Heavy bacterial infections
D. ____Unconjugated bilirubin 43. A positive nitrite test and a negative leukocyte esterase
35. The principle of the reagent strip test for bilirubin test is an indication of a:
is the: A. Dilute random specimen
A. Diazo reaction B. Specimen with lysed leukocytes
B. Ehrlich reaction C. Vaginal yeast infection
C. Greiss reaction D. Specimen older than 2 hours
D. Peroxidase reaction 44. All of the following can be detected by the leukocyte
36. An elevated urine bilirubin with a normal urobilinogen esterase reaction except:
is indicative of: A. Neutrophils
A. Cirrhosis of the liver B. Eosinophils
B. Hemolytic disease C. Lymphocytes
C. Hepatitis D. Basophils
D. Biliary obstruction 45. Screening tests for urinary infection combine the leuko-
cyte esterase test with the test for:
37. The primary cause of a false-negative bilirubin reaction is:
A. pH
A. Highly pigmented urine
B. Nitrite
B. Specimen contamination
C. Protein
C. Specimen exposure to light
D. Blood
D. Excess conjugated bilirubin
46. The principle of the leukocyte esterase reagent strip test
38. The purpose of the special mat supplied with the Ictotest
uses a:
tablets is that:
A. Peroxidase reaction
A. Bilirubin remains on the surface of the mat.
B. Double indicator reaction
B. It contains the dye needed to produce color.
C. Diazo reaction
C. It removes interfering substances.
D. Dye-binding technique
D. Bilirubin is absorbed into the mat.
47. The principle of the reagent strip test for specific gravity
39. The reagent in the Multistix reaction for urobilinogen is: uses the dissociation constant of a(n):
A. A diazonium salt A. Diazonium salt
B. Tetramethylbenzidine B. Indicator dye
C. p-Dimethylaminobenzaldehyde C. Polyelectrolyte
D. Hoesch reagent D. Enzyme substrate
40. The primary problem with urobilinogen tests using 48. A specific gravity of 1.005 would produce the reagent
Ehrlich reagent is: strip color:
A. Positive reactions with porphobilinogen A. Blue
B. Lack of specificity B. Green
C. Positive reactions with Ehrlich’s reactive substances C. Yellow
D. All of the above D. Red
41. The reagent strip test for nitrite uses the: 49. Reagent strip–specific gravity readings are affected by:
A. Greiss reaction A. Glucose
B. Ehrlich reaction B. Radiographic dye
C. Peroxidase reaction C. Alkaline urine
D. Pseudoperoxidase reaction D. All of the above
3920_Ch05_071-098 23/01/14 10:16 AM Page 96

96 Part Two | Urinalysis

Case Studies and Clinical Situations


1. A patient taken to the emergency department after an SP. GRAVITY: 1.020 BILIRUBIN: Negative
episode of syncope has a fasting blood glucose level
pH: 6.0 UROBILINOGEN: 8 EU
of 450 mg/dL. Results of the routine urinalysis are as
follows: PROTEIN: Negative NITRITE: Negative
COLOR: Yellow KETONES: 2+ GLUCOSE: Negative LEUKOCYTES: Negative
CLARITY: Clear BLOOD: Negative a. Would these results be indicative of hematuria or
SP. GRAVITY: 1.015 BILIRUBIN: Negative hemoglobinuria?
pH: 5.0 PROTEIN-LOW: 15 mg/dL b. Correlate the patient’s condition with the urobilino-
PROTEIN-HIGH: NITRITE: Negative gen result.
30 mg/dL c. Why is the urine bilirubin result negative in this
GLUCOSE: 250 mg/dL LEUKOCYTES: Negative jaundiced patient?
CREATININE: 200 mg/dL d. Would this method also measure urine porphyrins?
Why or why not?
a. Explain the correlation between the patient’s blood
and urine glucose results. 4. A female patient arrives at the outpatient clinic with
b. What is the most probable metabolic disorder asso- symptoms of lower back pain and urinary frequency
ciated with this patient? with a burning sensation. She is a firm believer in the
curative powers of vitamins. She has tripled her usual
c. Considering the patient’s condition, what is the sig-
dosage of vitamins in an effort to alleviate her symp-
nificance of the patient’s protein to creatinine ratio
toms; however, the symptoms have persisted. She is
result?
given a sterile container and asked to collect a mid-
d. If the patient in this study had a normal blood stream clean-catch urine specimen. Results of this
glucose level and normal protein and creatinine routine urinalysis are as follows:
results, to what would the urinary glucose be
attributed? COLOR: Dark yellow KETONES: Negative

2. Results of a urinalysis performed on a patient sched- CLARITY: Hazy BLOOD: Negative


uled for gallbladder surgery are as follows: SP. GRAVITY: 1.012 BILIRUBIN: Negative
COLOR: Amber KETONES: Negative pH: 7.0 UROBILINOGEN: Normal
CLARITY: Hazy BLOOD: Negative PROTEIN: Trace NITRITE: Negative
SP. GRAVITY: 1.022 BILIRUBIN: Moderate GLUCOSE: Negative LEUKOCYTES: 1+
pH: 6.0 UROBILINOGEN: Normal Microscopic
PROTEIN: Negative NITRITE: Negative 8 TO 12 RBC/HPF Heavy bacteria
GLUCOSE: Negative LEUKOCYTES: Negative 40 TO 50 WBC/HPF Moderate squamous epithelial
a. What would be observed if this specimen were cells
shaken? a. What discrepancies between the chemical and mi-
b. Explain the correlation between the patient’s sched- croscopic test results are present? State and explain
uled surgery and the normal urobilinogen. a possible reason for each discrepancy.
c. If blood were drawn from this patient, how might b. What additional chemical tests could be affected by
the appearance of the serum be described? the patient’s vitamin dosage? Explain the principle
d. What special handling is needed for serum and of the interference.
urine specimens from this patient? c. Discuss the urine color and specific gravity results
with regard to correlation and give a possible cause
3. Results of a urinalysis on a very anemic and jaundiced
for any discrepancy.
patient are as follows:
d. State three additional reasons not previously given
COLOR: Red KETONES: Negative for a negative nitrite test in the presence of in-
CLARITY: Clear BLOOD: Large creased bacteria.
3920_Ch05_071-098 23/01/14 10:16 AM Page 97

Chapter 5 | Chemical Examination of Urine 97

5. Results of a urinalysis collected following practice from SP. GRAVITY: 1.017 BILIRUBIN: Negative
a 20-year-old college athlete are as follows:
pH: 6.5 UROBILINOGEN: 0.4 EU
COLOR: Dark yellow KETONES: Negative PROTEIN: Trace NITRITE: Negative
CLARITY: Hazy BLOOD: 1+ GLUCOSE: Negative LEUKOCYTES: Negative
SP. GRAVITY: 1.029 BILIRUBIN: Negative a. Would hematuria be suspected in this specimen?
Why or why not?
pH: 6.5 UROBILINOGEN: 1 EU
b. What is the most probable cause of the positive
PROTEIN: 2+ NITRITE: Negative blood reaction?
GLUCOSE: Negative LEUKOCYTES: Negative c. What is the source of the substance causing the pos-
itive blood reaction and the name of the condition?
The physician requests that the athlete collect another
d. Would this patient be monitored for changes in
specimen in the morning prior to classes and practice.
renal function? Why or why not?
a. What is the purpose of the second sample?
7. Considering the correct procedures for care, technique,
b. What changes would you expect in the second and quality control for reagent strips, state a possible
sample? cause for each of the following scenarios.
c. Is the proteinuria present in the first sample of prer- a. The urinalysis supervisor notices that an unusually
enal, renal, or postrenal origin? large number of reagent strips are becoming discol-
6. A construction worker is pinned under collapsed scaf- ored before the expiration date has been reached.
folding for several hours prior to being taken to the b. A physician’s office is consistently reporting positive
emergency room. His abdomen and upper legs are se- nitrite test results with negative LE test results.
verely bruised, but no fractures are detected. A speci- c. A student’s results for reagent strip blood and LE
men for urinalysis obtained by catheterization has the are consistently lower than those of the laboratory
following results: staff.
d. One morning the urinalysis laboratory was report-
COLOR: Red-brown KETONES: Negative
ing results that were frequently questioned by
CLARITY: Clear BLOOD: 4+ physicians.
3920_Ch06_099-146 23/01/14 10:19 AM Page 141

Chapter 6 | Microscopic Examination of Urine 141

Study Questions
1. Macroscopic screening of urine specimens is used to: 8. Which of the following are reported as number per lpf?
A. Provide results as soon as possible A. RBCs
B. Predict the type of urinary casts present B. WBCs
C. Increase cost-effectiveness of urinalysis C. Crystals
D. Decrease the need for polarized microscopy D. Casts
2. Variations in the microscopic analysis of urine include all 9. The Sternheimer-Malbin stain is added to urine sediments
of the following except: to do all of the following except:
A. Preparation of the urine sediment A. Increase visibility of sediment constituents
B. Amount of sediment analyzed B. Change the constituents’ refractive index
C. Method of reporting C. Decrease precipitation of crystals
D. Identification of formed elements D. Delineate constituent structures
3. All of the following can cause false-negative microscopic 10. Nuclear detail can be enhanced by:
results except: A. Prussian blue
A. Braking the centrifuge B. Toluidine blue
B. Failing to mix the specimen C. Acetic acid
C. Dilute alkaline urine D. Both B and C
D. Using midstream clean-catch specimens
11. Which of the following lipids is/are stained by Sudan III?
4. The two factors that determine relative centrifugal force are: A. Cholesterol
A. Radius of rotor head and rpm B. Neutral fats
B. Radius of rotor head and time of centrifugation C. Triglycerides
C. Diameter of rotor head and rpm D. Both B and C
D. RPM and time of centrifugation
12. Which of the following lipids is/are capable of polarizing
5. When using the glass slide and cover-slip method, which of light?
the following might be missed if the cover slip is overflowed? A. Cholesterol
A. Casts B. Neutral fats
B. RBCs C. Triglycerides
C. WBCs D. Both A and B
D. Bacteria
13. The purpose of the Hansel stain is to identify:
6. Initial screening of the urine sediment is performed using A. Neutrophils
an objective power of:
B. Renal tubular cells
A. 4× C. Eosinophils
B. 10× D. Monocytes
C. 40×
14. Crenated RBCs are seen in urine that is:
D. 100×
A. Hyposthenuric
7. Which of the following should be used to reduce light B. Hypersthenuric
intensity in bright-field microscopy?
C. Highly acidic
A. Centering screws
D. Highly alkaline
B. Aperture diaphragm
C. Rheostat
D. Condenser aperture diaphragm
3920_Ch06_099-146 23/01/14 10:19 AM Page 142

142 Part Two | Urinalysis

15. Differentiation among RBCs, yeast, and oil droplets may 23. Forms of transitional epithelial cells include all of the
be accomplished by all of the following except: following except:
A. Observation of budding in yeast cells A. Spherical
B. Increased refractility of oil droplets B. Caudate
C. Lysis of yeast cells by acetic acid C. Convoluted
D. Lysis of RBCs by acetic acid D. Polyhedral
16. A finding of dysmorphic RBCs is indicative of: 24. Increased transitional cells are indicative of:
A. Glomerular bleeding A. Catheterization
B. Renal calculi B. Malignancy
C. Traumatic injury C. Pyelonephritis
D. Coagulation disorders D. Both A and B
17. Leukocytes that stain pale blue with Sternheimer-Malbin 25. A primary characteristic used to identify renal tubular
stain and exhibit brownian movement are: epithelial cells is:
A. Indicative of pyelonephritis A. Elongated structure
B. Basophils B. Centrally located nucleus
C. Mononuclear leukocytes C. Spherical appearance
D. Glitter cells D. Eccentrically located nucleus
18. Mononuclear leukocytes are sometimes mistaken for: 26. Following an episode of hemoglobinuria, RTE cells may
A. Yeast cells contain:
B. Squamous epithelial cells A. Bilirubin
C. Pollen grains B. Hemosiderin granules
D. Renal tubular cells C. Porphobilinogen
D. Myoglobin
19. When pyuria is detected in a urine sediment, the slide
should be carefully checked for the presence of: 27. The predecessor of the oval fat body is the:
A. RBCs A. Histiocyte
B. Bacteria B. Urothelial cell
C. Hyaline casts C. Monocyte
D. Mucus D. Renal tubular cell

20. Transitional epithelial cells are sloughed from the: 28. A structure believed to be an oval fat body produced a
Maltese cross formation under polarized light but does not
A. Collecting duct
stain with Sudan III. The structure:
B. Vagina
A. Contains cholesterol
C. Bladder
B. Is not an oval fat body
D. Proximal convoluted tubule
C. Contains neutral fats
21. The largest cells in the urine sediment are: D. Is contaminated with immersion oil
A. Squamous epithelial cells
29. The finding of yeast cells in the urine is commonly asso-
B. Urothelial epithelial cells ciated with:
C. Cuboidal epithelial cells A. Cystitis
D. Columnar epithelial cells B. Diabetes mellitus
22. A clinically significant squamous epithelial cell is the: C. Pyelonephritis
A. Cuboidal cell D. Liver disorders
B. Clue cell
C. Caudate cell
D. Columnar cell
3920_Ch06_099-146 23/01/14 10:20 AM Page 143

Chapter 6 | Microscopic Examination of Urine 143

30. The primary component of urinary mucus is: 38. The presence of fatty casts is associated with:
A. Bence Jones protein A. Nephrotic syndrome
B. Microalbumin B. Crush injuries
C. Uromodulin C. Diabetes mellitus
D. Orthostatic protein D. All of the above
31. The majority of casts are formed in the: 39. Nonpathogenic granular casts contain:
A. Proximal convoluted tubules A. Cellular lysosomes
B. Ascending loop of Henle B. Degenerated cells
C. Distal convoluted tubules C. Protein aggregates
D. Collecting ducts D. Gram-positive cocci
32. Cylindruria refers to the presence of: 40. All of the following are true about waxy casts except they:
A. Cylindrical renal tubular cells A. Represent extreme urine stasis
B. Mucus-resembling casts B. May have a brittle consistency
C. Hyaline and waxy casts C. Require staining to be visualized
D. All types of casts D. Contain degenerated granules
33. A person submitting a urine specimen following a stren- 41. Observation of broad casts represents:
uous exercise routine can normally have all of the follow-
ing in the sediment except: A. Destruction of tubular walls
A. Hyaline casts B. Dehydration and high fever
B. Granular casts C. Formation in the collecting ducts
C. RBC casts D. Both A and C
D. WBC casts 42. All of the following contribute to urinary crystals forma-
tion except:
34. Prior to identifying an RBC cast, all of the following
should be observed except: A. Protein concentration
A. Free-floating RBCs B. pH
B. Intact RBCs in the cast C. Solute concentration
C. Presence of a cast matrix D. Temperature
D. A positive reagent strip blood reaction 43. The most valuable initial aid for identifying crystals in a
35. WBC casts are primarily associated with: urine specimen is:
A. Pyelonephritis A. pH
B. Cystitis B. Solubility
C. Glomerulonephritis C. Staining
D. Viral infections D. Polarized microscopy

36. The shape of the RTE cell associated with renal tubular 44. Crystals associated with severe liver disease include all of
epithelial casts is primarily: the following except:
A. Elongated A. Bilirubin
B. Cuboidal B. Leucine
C. Round C. Cystine
D. Columnar D. Tyrosine
37. When observing RTE casts, the cells are primarily: 45. All of the following crystals routinely polarize except:
A. Embedded in a clear matrix A. Uric acid
B. Embedded in a granular matrix B. Cholesterol
C. Attached to the surface of a matrix C. Radiographic dye
D. Stained by components of the urine filtrate D. Cystine
3920_Ch06_099-146 23/01/14 10:20 AM Page 144

144 Part Two | Urinalysis

46. Casts and fibers can usually be differentiated using: 50. Match the following types of microscopy with their
A. Solubility characteristics descriptions:
B. Patient history ____ Bright-field 1. Indirect light is reflected
off the object
C. Polarized light
____ Phase 2. Objects split light into two
D. Fluorescent light
beams
47. Match the following crystals seen in acidic urine with their ____ Polarized 3. Low refractive index
description/identifying characteristics: objects may be overlooked
____ Amorphous urates 1. Envelopes ____ Dark-field 4. Three-dimensional images
____ Uric acid 2. Thin needles ____Fluorescent 5. Forms halo of light around
____ Calcium oxalate 3. Yellow-brown, object
monohydrate whetstone ____Interference 6. Detects electrons
____ Calcium oxalate 4. Pink sediment contrast emitted from objects
dihydrate 7. Detects specific wavelengths
5. Ovoid of light emitted from objects

48. Match the following crystals seen in alkaline urine with


their description/identifying characteristics:
____ Triple phosphate 1. Yellow granules
____ Amorphous phosphate 2. Thin prisms
____ Calcium phosphate 3. “Coffin lids”
____ Ammonium biurate 4. Dumbbell shape
____ Calcium carbonate 5. White precipitate
6. Thorny apple
49. Match the following abnormal crystals with their
description/identifying characteristics:
____ Cystine 1. Bundles following
refrigeration
____ Tyrosine 2. Highly alkaline pH
____ Cholesterol 3. Bright yellow clumps
____ Leucine 4. Hexagonal plates
____ Ampicillin 5. Flat plates, high
specific gravity
____ Radiographic dye 6. Concentric circles,
radial striations
____ Bilirubin 7. Notched corners
8. Fine needles seen in
liver disease
3920_Ch06_099-146 23/01/14 10:20 AM Page 145

Chapter 6 | Microscopic Examination of Urine 145

Case Studies and Clinical Situations


1. An 85-year-old woman with diabetes and a broken hip Additional testing detects a superinfection with delta
has been confined to bed for the past 3 months. Results hepatitis virus and decreased renal concentrating ability.
of an ancillary blood glucose test are 250 mg/dL, and her Urinalysis results are as follows:
physician orders additional blood tests and a routine COLOR: Amber KETONES: Negative
urinalysis. The urinalysis report is as follows:
CLARITY: Hazy BLOOD: Negative
COLOR: Pale yellow KETONES: Negative SP. GRAVITY: 1.011 BILIRUBIN: Large
CLARITY: Hazy BLOOD: Moderate pH: 7.0 UROBILINOGEN: 4.0 EU
SP. GRAVITY: 1.020 BILIRUBIN: Negative PROTEIN: 2+ NITRITE: Negative
pH: 5.5 UROBILINOGEN: Normal GLUCOSE: Negative LEUKOCYTES: Negative
PROTEIN: Trace NITRITE: Negative Microscopic:
GLUCOSE: 100 mg/dL LEUKOCYTES: 2+ 2 to 4 WBCs/hpf 1 to 2 hyaline casts/lpf
Microscopic: 1 to 3 RBCs/hpf 1 to 2 granular casts/lpf
20 to 25 WBCs/hpf 2 to 4 bile-stained RTE
Many yeast cells and hyphae cells/hpf
a. Why are yeast infections common in patients with 0 to 1 RTE casts/lpf
diabetes mellitus? 0 to 1 bile-stained waxy
b. With a blood glucose level of 250 mg/dL, should casts/lpf
glucose be present in the urine? Why or why not? a. Based on the urinalysis results, in what area of the
c. Is there a discrepancy between the negative nitrite and nephron is damage occurring?
the positive leukocyte esterase results? Explain your b. Is this consistent with the patient’s primary diagnosis?
answer. Explain your answer.
d. What is the major discrepancy between the chemical c. What is causing the RTE cells to be bile stained?
and microscopic results? d. Why is the urobilinogen level elevated?
e. Considering the patient’s history, what is the most e. State a disorder in which the urobilinogen level would
probable cause for the discrepancy? be elevated, but the bilirubin result would be negative.
2. A medical technology student training in a newly reno- 4. A 30-year-old woman being treated for a UTI brings a
vated STAT laboratory is having difficulty performing a urine specimen to the Employee Health Clinic at 4:00 p.m.
microscopic urinalysis. Reagent strip testing indicates the The nurse on duty tells her that the specimen will be re-
presence of moderate blood and leukocytes, but the stu- frigerated and tested by the technologist the next morning.
dent is also observing some large unusual objects resem- The technologist has difficulty interpreting the color of the
bling crystals and possible casts. The student is also reagent strip tests and reports only the following results:
having difficulty keeping all of the constituents in focus COLOR: Amber CLARITY: Slightly cloudy
at the same time.
Microscopic:
a. Why is the student having difficulty focusing?
3 to 5 RBCs/hpf
b. What is a possible cause of the unusual microscopic
8 to 10 WBCs/hpf
constituents?
Moderate bacteria
c. Should the student be concerned about the unusual
microscopic constituents? Explain your answer. Moderate colorless crystals appearing in bundles
d. What microscopy technique could be used to aid in a. What could have caused the technologist to have
differentiating a cast and an artifact? difficulty interpreting the reagent strip results?
b. Could this specimen produce a yellow foam when
3. A prisoner sentenced to 10 years for selling illegal drugs
shaken?
develops jaundice, lethargy, and hepatomegaly.
A test for hepatitis B surface antigen is positive, and the c. What could the technologist do to aid in the identifi-
patient is placed in the prison infirmary. When his condi- cation of the crystals?
tion appears to worsen and a low urinary output is ob- d. What is the probable identification of the colorless
served, the patient is transferred to a local hospital. crystals?
3920_Ch06_099-146 23/01/14 10:20 AM Page 146

146 Part Two | Urinalysis

5. A 2-year-old left unattended in the garage for 5 minutes SP. GRAVITY: 1.030 BILIRUBIN: Negative
is suspected of ingesting antifreeze (ethylene glycol). The pH: 5.5 UROBILINOGEN: Normal
urinalysis has a pH of 6.0 and is negative on the chemical
PROTEIN: 2+ NITRITE: Negative
examination. Two distinct forms of crystals are observed
in the microscopic examination. GLUCOSE: Negative LEUKOCYTE: Negative
a. What type of crystals would you expect to be present? Microscopic:
b. What is the other form of this crystal? 0 to 3 WBCs/hpf
c. Describe the two forms. 0 to 4 hyaline casts/lpf
d. Which form would you expect to be predominant? 0 to 3 granular casts/lpf
Few squamous epithelial cells
6. A female patient comes to the outpatient clinic with
symptoms of UTI. She brings a urine specimen with her. a. Are these results of clinical significance?
Results of the routine analysis performed on this speci- b. Explain the discrepancy between the chemical and
men are as follows: microscopic blood results.
COLOR: Yellow KETONES: Negative c. What is the probable cause of the granular casts?
CLARITY: Hazy BLOOD: Small 8. As supervisor of the urinalysis section, you are reviewing
SP. GRAVITY: 1.015 BILIRUBIN: Negative results. State why or why not each of the following results
pH: 9.0 UROBILINOGEN: Normal would concern you.
PROTEIN: Negative NITRITE: Negative a. The presence of waxy casts and a negative protein in
urine from a 6–month-old girl
GLUCOSE: Negative LEUKOCYTE: 2+
b. Increased transitional epithelial cells in a specimen
Microscopic:
obtained following cystoscopy
1 to 3 RBCs/hpf Heavy bacteria
c. Tyrosine crystals in a specimen with a negative
8 to 10 WBCs/hpf Moderate squamous bilirubin test result
epithelial cells
d. Cystine crystals in a specimen from a patient diag-
a. What discrepancies are present between the chemical nosed with gout
and microscopic test results?
e. Cholesterol crystals in urine with a specific gravity
b. State a reason for the discrepancies. greater than 1.040
c. Identify a chemical result in the urinalysis that con- f. Trichomonas vaginalis in a male urine specimen
firms your reason for the discrepancies.
g. Amorphous urates and calcium carbonate crystals in
d. What course of action should the laboratory take to a specimen with a pH of 7.0
obtain accurate results for this patient?
7. A high school student is taken to the emergency room
with a broken leg that occurred during a football game.
The urinalysis results are as follows:
COLOR: Dark yellow KETONES: Negative
CLARITY: Hazy BLOOD: Moderate
3920_Ch07_147-162 23/01/14 10:21 AM Page 158

158 Part Two | Urinalysis

Study Questions
1. Most glomerular disorders are caused by: 8. The highest levels of proteinuria are seen with:
A. Sudden drops in blood pressure A. Alport syndrome
B. Immunologic disorders B. Diabetic nephropathy
C. Exposure to toxic substances C. IgA nephropathy
D. Bacterial infections D. Nephrotic syndrome

2. Dysmorphic RBC casts would be a significant finding with 9. Ischemia frequently produces:
all of the following except: A. Acute renal tubular necrosis
A. Goodpasture syndrome B. Minimal change disorder
B. Acute glomerulonephritis C. Renal glycosuria
C. Chronic pyelonephritis D. Goodpasture’s syndrome
D. Henoch-Schönlein purpura 10. A disorder associated with polyuria and low specific
3. Occasional episodes of macroscopic hematuria over periods gravity is:
of 20 or more years are seen with: A. Renal glucosuria
A. Crescentic glomerulonephritis B. Minimal change disease
B. IgA nephropathy C. Nephrogenic diabetes insipidus
C. Nephrotic syndrome D. Focal segmental glomerulosclerosis
D. Wegener granulomatosis 11. An inherited disorder producing a generalized defect in
tubular reabsorption is:
4. Antiglomerular basement membrane antibody is seen with:
A. Alport syndrome
A. Wegener granulomatosis
B. Acute interstitial nephritis
B. IgA nephropathy
C. Fanconi syndrome
C. Goodpasture syndrome
D. Renal glycosuria
D. Diabetic nephropathy
12. A teenage boy who develops gout in his big toe and has a
5. Antineutrophilic cytoplasmic antibody is diagnostic for: high serum uric acid should be monitored for:
A. IgA nephropathy A. Fanconi syndrome
B. Wegener granulomatosis B. Renal calculi
C. Henoch-Schönlein purpura C. Uromodulin-associated kidney disease
D. Goodpasture syndrome D. Chronic interstitial nephritis
6. Respiratory and renal symptoms are associated with all of 13. The only protein produced by the kidney is:
the following except: A. Albumin
A. IgA nephropathy B. Uromodulin
B. Wegener granulomatosis C. Uroprotein
C. Henoch-Schönlein purpura D. Globulin
D. Goodpasture syndrome
14. The presence of renal tubular epithelial cells and casts is
7. The presence of fatty casts is associated with all of the fol- an indication of:
lowing except: A. Acute interstitial nephritis
A. Nephrotic syndrome B. Chronic glomerulonephritis
B. Focal segmental glomerulosclerosis C. Minimal change disease
C. Nephrogenic diabetes insipidus D. Acute tubular necrosis
D. Minimal change disease
3920_Ch07_147-162 23/01/14 10:21 AM Page 159

Chapter 7 | Renal Disease 159

15. Differentiation between cystitis and pyelonephritis is aided 18. Prerenal acute renal failure could be caused by:
by the presence of: A. Massive hemorrhage
A. WBC casts B. Acute tubular necrosis
B. RBC casts C. Acute interstitial nephritis
C. Bacteria D. Malignant tumors
D. Granular casts
19. The most common composition of renal calculi is:
16. The presence of WBCs and WBC casts with no bacteria is A. Calcium oxalate
indicative of:
B. Magnesium ammonium phosphate
A. Chronic pyelonephritis
C. Cystine
B. Acute tubular necrosis
D. Uric acid
C. Acute interstitial nephritis
20. Urinalysis on a patient with severe back pain being eval-
D. Both B and C
uated for renal calculi would be most beneficial if it
17. End-stage renal disease is characterized by all of the showed:
following except: A. Heavy proteinuria
A. Hypersthenuria B. Low specific gravity
B. Isosthenuria C. Uric acid crystals
C. Azotemia D. Microscopic hematuria
D. Electrolyte imbalance

Case Studies and Clinical Situations


1. A 14-year-old boy who has recently recovered from a sore e. What is the expected prognosis of this patient?
throat develops edema and hematuria. Significant labora- f. If the above urinalysis results were seen in a 5-year-old
tory results include a BUN of 30 mg/dL (normal 8 to boy who has developed a red, patchy rash following
23 mg/dL) and a positive group A streptococcal antibody recovery from a respiratory infection, what disorder
test. Results of a urinalysis are as follows: would you suspect?
Color: Red Ketones: Negative 2. B.J. is a seriously ill 40-year-old man with a history of
Clarity: Cloudy Blood: Large several episodes of macroscopic hematuria in the past
Sp. gravity: 1.020 Bilirubin: Negative 20 years. The episodes were associated with exercise or
stress. Until recently the macroscopic hematuria had
pH: 5.0 Urobilinogen: Normal spontaneously reverted to asymptomatic microscopic
Protein: 3+ Nitrite: Negative hematuria. Significant laboratory results include a BUN of
Glucose: Negative Leukocyte: Trace 80 mg/dL (normal 8 to 23 mg/dL), serum creatinine of
4.5 mg/dL (normal 0.6 to 1.2 mg/dL), creatinine clearance
Microscopic:
of 20 mL/min (normal 107 to 139 mL/min), serum calcium
100 RBCs/hpf—many dysmorphic forms of 8.0 mg/dL (normal 9.2 to 11.0 mg/dL), serum phospho-
5–8 WBCs/hpf rus of 6.0 mg/dL (normal 2.3 to 4.7 mg/dL), and an ele-
0–2 granular casts/lpf vated level of serum IgA. Results of a routine urinalysis
are as follows:
0–1 RBC casts/lpf
Color: Red Ketones: Negative
a. What disorder do these results and history
Clarity: Slightly cloudy Blood: Large
indicate?
Sp. gravity: 1.010 Bilirubin: Negative
b. What specific characteristic was present in the
organism causing the sore throat? pH: 6.5 Urobilinogen: Normal
Protein: 300 mg/dL Nitrite: Negative
c. What is the significance of the dysmorphic RBCs?
Glucose: 250 mg/dL Leukocyte: Trace
d. Are the WBCs significant? Why or why not?
3920_Ch07_147-162 23/01/14 10:21 AM Page 160

160 Part Two | Urinalysis

Microscopic: and creatinine results and markedly decreased total


>100 RBCs/hpf 2–4 hyaline casts/lpf protein and albumin values. Urinalysis results are as
follows:
8–10 WBCs/hpf 1–5 granular casts/lpf
Color: Yellow Ketones: Negative
0–2 waxy casts/lpf 0–2 broad waxy
Clarity: Hazy Blood: Small
a. What specific disease do the patient’s laboratory
results and history suggest? Sp. gravity: 1.018 Bilirubin: Negative
b. Which laboratory result is most helpful in diagnosing pH: 6.5 Urobilinogen: Normal
this disease? Protein: 4+ Nitrite: Negative
c. What additional diagnosis does his current condition Glucose: Negative Leukocyte: Negative
suggest? Microscopic:
d. What is the significance of the positive result for urine 10–15 RBCs/hpf 0–1 hyaline casts/lpf
glucose?
0–4 WBCs/hpf 0–2 granular casts/lpf
e. Is the specific gravity significant? Why or why not?
Moderate fat droplets 0–1 oval fat bodies/hpf
f. What is the significance of the waxy casts?
a. What disorder do the patient history, physical
3. A 45-year-old woman is recovering from injuries received appearance, and laboratory results suggest?
in an automobile accident that resulted in her being taken b. What other renal disorders produce similar
to the emergency department with severe hypotension. urinalysis results?
She develops massive edema. Significant laboratory re-
c. What is the expected prognosis for this patient?
sults include a BUN of 30 mg/dL (normal 8 to 23 mg/dL),
cholesterol of 400 mg/dL (normal 150 to 240 mg/dL), 5. A 32-year-old construction worker experiences
triglycerides of 840 mg/dL (normal 10 to 190 mg/dL), respiratory difficulty followed by the appearance
serum protein of 4.5 mg/dL (normal 6.0 to 7.8 mg/dL), of blood-streaked sputum. He delays visiting a
albumin of 2.0 mg/dL (normal 3.2 to 4.5 mg/dL), and physician until symptoms of extreme fatigue and
a total urine protein of 3.8 g/d (normal 100 mg/d). red urine are present. A chest radiograph shows pul-
Urinalysis results are as follows: monary infiltration, and sputum culture is negative
Color: Yellow Ketones: Negative for pathogens. Blood test results indicate anemia,
increased BUN and creatinine, and the presence
Clarity: Cloudy Blood: Moderate
of antiglomerular basement membrane antibody.
Sp. gravity: 1.015 Bilirubin: Negative Urinalysis results are as follows:
pH: 6.0 Urobilinogen: Normal Color: Red Ketones: Negative
Protein: 4+ Nitrite: Negative Clarity: Cloudy Blood: Large
Glucose: Negative Leukocyte: Negative Sp. gravity: 1.015 Bilirubin: Negative
Microscopic: ph: 6.0 Urobilinogen: Normal
15–20 RBCs/hpf Moderate free fat droplets Protein: 3+ Nitrite: Negative
0–2 granular casts/lpf
Glucose: Negative Leukocyte: Trace
0–5 WBCs/hpf Moderate cholesterol
Microscopic:
0–2 fatty casts/lpf crystals
100 RBCs/hpf 0–3 hyaline casts/lpf
0–2 oval fat bodies/hpf
10–15 WBCs/hpf 0–3 granular casts/lpf
a. What renal disorder do these results suggest?
0–2 RBCs casts/lpf
b. How does the patient’s history relate to this
disorder? a. What disorder do the laboratory results suggest?
c. What physiologic mechanism accounts for the b. How is this disorder affecting the glomerulus?
massive proteinuria? c. If the antiglomerular membrane antibody test is nega-
d. What is the relationship of the proteinuria to the tive, what disorder might be considered?
edema? d. What is the diagnostic test for this disorder?
e. What mechanism produces the oval fat bodies? e. By what mechanism does this disorder affect the
glomerulus?
4. A routinely active 4-year-old boy becomes increasingly
less active after receiving several preschool immuniza- 6. A 25-year-old pregnant woman comes to the outpatient
tions. His pediatrician observes noticeable puffiness clinic with symptoms of lower back pain, urinary
around the eyes. A blood test shows normal BUN frequency, and a burning sensation when voiding.
3920_Ch07_147-162 23/01/14 10:21 AM Page 161

Chapter 7 | Renal Disease 161

Her pregnancy has been normal up to this time. She Microscopic:


is given a sterile container and asked to collect a 6–10 RBCs/hpf Many bacteria
midstream clean-catch urine specimen. Routine 0–2 WBC casts/lpf
urinalysis results are as follows:
>100 WBCs/hpf Moderate birefringent,
Color: Pale yellow Ketones: Negative 0–1 bacterial casts/lpf flat crystals
Clarity: Hazy Blood: Small a. What diagnostic procedure was performed on the pa-
Sp. gravity: 1.005 Bilirubin: Negative tient that could account for the differences in the two
pH: 8.0 Urobilinogen: Normal urinalysis results?
Protein: Trace Nitrite: Positive b. Considering the patient’s age and history, what is the
most probable diagnosis?
Glucose: Negative Leukocyte: 2+
c. What microscopic constituent is most helpful to this
Microscopic:
diagnosis?
6–10 RBCs/hpf Heavy bacteria
d. What is the most probable cause of this disorder?
40–50 WBCs/hpf Moderate squamous
e. How can the presence of the bacterial cast be
epithelial cells
confirmed?
a. What is the most probable diagnosis for this
f. What is the most probable source of the crystals
patient?
present in the sediment?
b. What is the correlation between the color and the
g. Without surgical intervention, what is the patient’s
specific gravity?
prognosis?
c. What is the significance of the blood and protein
tests? 8. A 35-year-old patient being treated for a sinus infection
with methicillin develops fever, a skin rash, and edema.
d. Is this specimen suitable for the appearance of
Urinalysis results are as follows:
glitter cells? Explain your answer.
Color: Dark yellow Ketones: Negative
e. What other population is at a high risk for
developing this condition? Clarity: Cloudy Blood: Moderate
f. What disorder might develop if this disorder is not Sp. gravity: 1.012 Bilirubin: Negative
treated? pH: 6.0 Urobilinogen: Normal
7. A 10-year-old patient with a history of recurrent UTIs is Protein: 3+ Nitrite: Negative
admitted to the hospital for diagnostic tests. Initial urinal- Glucose: Negative Leukocyte: 2+
ysis results are as follows: Microscopic:
Color: Yellow Ketone: Negative 20–30 RBCs/hpf 1–2 WBC casts/lpf
Clarity: Cloudy Blood: Small >100 WBCs/hpf 1–2 granular casts/lpf
Sp. gravity: 1.025 Bilirubin: Negative After receiving the urinalysis report, the physician
pH: 8.0 Urobilinogen: Normal orders a test for urinary eosinophils. The urinary
Protein: 2+ Nitrite: Positive eosinophil result is 10%.
Glucose: Negative Leukocyte: 2+ a. Is the urinary eosinophil result normal or
abnormal?
Microscopic:
b. What is the probable diagnosis for this patient?
6–10 RBCs/hpf 0–2 WBC casts/lpf Many
bacteria c. Discuss the significance of the increased WBCs
and WBC casts in the absence of bacteria.
>100 WBCs/hpf 0–1 bacterial casts/lpf with
clumps d. How can this condition be corrected?
A repeat urinalysis a day later has the following results: 9. Following surgery to correct a massive hemorrhage, a
Color: Yellow Ketones: Negative 55-year-old patient exhibits oliguria and edema. Blood
test results indicate increasing azotemia and electrolyte
clarity: Cloudy Blood: Small
imbalance. The glomerular filtration rate is 20 mL/min.
Sp. gravity: >1.035 bilirubin: Negative Urinalysis results are as follows:
pH: 7.5 Urobilinogen: Normal Color: Yellow Ketones: Negative
Protein: 2+ Nitrite: Positive Clarity: Cloudy Blood: Moderate
Glucose: Negative Leukocyte: 2+ Sp. gravity: 1.010 Bilirubin: Negative
3920_Ch07_147-162 23/01/14 10:21 AM Page 162

162 Part Two | Urinalysis

pH: 7.0 Urobilinogen: Normal and an amylase are normal. Results of a routine
Protein: 3+ Nitrite: Negative urinalysis are as follows:
Glucose: 2+ Leukocyte: Negative Color: Dark yellow Ketones: Negative
Microscopic: Clarity: Hazy Blood: Moderate
50–60 RBCs/hpf 2–3 granular casts/lpf Sp. gravity: 1.030 Bilirubin: Negative
3–6 WBCs/hpf 2–3 RTE cell casts/lpf pH: 5.0 Urobilinogen: Normal
3–4 RTE cells/hpf 0–1 waxy casts/lpf Protein: Trace Nitrite: Negative
0–1 broad granular casts/lpf Glucose: Negative Leukocytes: Negative
a. What diagnosis do the patient’s history and labora- Microscopic
tory results suggest? 15–20 RBCs/hpf
b. What is the most probable cause of the patient’s 0–2 WBCs/hpf
disorder? Is this considered to be of prerenal, renal, Few squamous epithelial cells
or postrenal origin?
a. What condition could these urinalysis results and
c. What is the significance of the specific gravity the patient’s symptoms represent?
result?
b. What would account for the crenated RBCs?
d. What is the significance of the RTE cells?
c. Is there a correlation between the urine color and
e. State two possible reasons for the presence of the specific gravity and the patient’s symptoms?
broad casts.
d. Based on the primary substance that causes this
10. A 40-year-old man develops severe back and abdomi- condition, what type of crystals might have been
nal pain after dinner. The pain subsides during the present?
night but recurs in the morning, and he visits his e. What changes will the patient be advised to make
family physician. Results of a complete blood count in his lifestyle to prevent future occurrences?
3920_Ch08_163-179 23/01/14 10:24 AM Page 175

Chapter 8 | Urine Screening for Metabolic Disorders 175

Study Questions
1. All states require newborn screening for PKU for early: 8. An overflow disorder that could produce a false-positive
A. Modifications of diet reaction with Clinitest procedure is:
B. Administration of antibiotics A. Cystinuria
C. Detection of diabetes B. Alkaptonuria
D. Initiation of gene therapy C. Indicanuria
D. Porphyrinuria
2. All of the following disorders can be detected by newborn
screening except: 9. A urine that turns black after sitting by the sink for
several hours could be indicative of:
A. Tyrosyluria
A. Alkaptonuria
B. MSUD
B. MSUD
C. Melanuria
C. Melanuria
D. Galactosemia
D. Both A and C
3. The best specimen for early newborn screening is a:
10. Ketonuria in a newborn is an indication of:
A. Timed urine specimen
A. MSUD
B. Blood specimen
B. Isovaleric acidemia
C. First morning urine specimen
C. Methylmalonic acidemia
D. Fecal specimen
D. All of the above
4. Abnormal urine screening tests categorized as an overflow
11. Urine from a newborn with MSUD will have a
disorder include all of the following except: significant:
A. Alkaptonuria A. Pale color
B. Galactosemia B. Yellow precipitate
C. Melanuria C. Milky appearance
D. Cystinuria D. Sweet odor
5. Which of the following disorders is not associated with the 12. Hartnup disease is a disorder associated with the metab-
phenylalanine-tyrosine pathway? olism of:
A. MSUD A. Organic acids
B. Alkaptonuria B. Tryptophan
C. Albinism C. Cystine
D. Tyrosinemia D. Phenylalanine
6. The least serious form of tyrosylemia is: 13. 5-HIAA is a degradation product of:
A. Immature liver function A. Heme
B. Type 1 B. Indole
C. Type 2 C. Serotonin
D. Type 3 D. Melanin

7. An overflow disorder of the phenylalanine-tyrosine path- 14. Elevated urinary levels of 5-HIAA are associated with:
way that would produce a positive reaction with the reagent A. Carcinoid tumors
strip test for ketones is: B. Hartnup disease
A. Alkaptonuria C. Cystinuria
B. Melanuria D. Platelet disorders
C. MSUD
D. Tyrosyluria
3920_Ch08_163-179 23/01/14 10:24 AM Page 176

176 Part Two | Urinalysis

15. False-positive levels of 5-HIAA can be caused by a diet 22. Which of the following specimens can be used for
high in: porphyrin testing?
A. Meat A. Urine
B. Carbohydrates B. Blood
C. Starch C. Feces
D. Bananas D. All of the above
16. Place the appropriate letter in front of the following 23. The two stages of heme formation affected by lead
statements. poisoning are:
A. Cystinuria A. Porphobilinogen and uroporphyrin
B. Cystinosis B. Aminolevulinic acid and porphobilinogen
____ IEM C. Coproporphyrin and protoporphyrin
____ Inherited disorder of tubular reabsorption D. Aminolevulinic acid and protoporphyrin
____ Fanconi syndrome 24. Hurler, Hunter, and Sanfilippo syndromes are hereditary
____ Cystine deposits in the cornea disorders affecting metabolism of:
____ Early renal calculi formation A. Porphyrins
17. Blue diaper syndrome is associated with: B. Purines
A. Lesch-Nyhan syndrome C. Mucopolysaccharides
B. Phenylketonuria D. Tryptophan
C. Cystinuria 25. Many uric acid crystals in a pediatric urine specimen may
D. Hartnup disease indicate:
A. Hurler syndrome
18. Homocystinuria is caused by failure to metabolize:
B. Lesch-Nyhan disease
A. Lysine
C. Melituria
B. Methionine
D. Sanfilippo syndrome
C. Arginine
D. Cystine 26. Deficiency of the GALT enzyme will produce a:
A. Positive Clinitest
19. The Ehrlich reaction will only detect the presence of:
B. Glycosuria
A. Uroporphyrin
C. Galactosemia
B. Porphobilinogen
D. Both A and C
C. Coproporphyrin
D. Protoporphyrin 27. Match the metabolic urine disorders with their classic
urine abnormalities.
20. Acetyl acetone is added to the urine before performing the
____ PKU A. Sulfur odor
Ehrlich test when checking for:
____ Indicanuria B. Sweaty feet odor
A. Aminolevulinic acid
____ Cystinuria C. Orange sand in diaper
B. Porphobilinogen
____ Alkaptonuria D. Mousy odor
C. Uroporphyrin
____ Lesch-Nyhan disease E. Black color
D. Coproporphyrin
____ Isovaleric acidemia F. Blue color
21. The classic urine color associated with porphyria is:
A. Dark yellow
B. Indigo blue
C. Pink
D. Port wine
3920_Ch08_163-179 23/01/14 10:24 AM Page 177

Chapter 8 | Urine Screening for Metabolic Disorders 177

Case Studies and Clinical Situations


1. A premature infant develops jaundice. Laboratory tests c. If amino acid chromatography was performed on
are negative for hemolytic disease of the newborn, but this specimen, what additional amino acids could be
the infant’s bilirubin level continues to rise. Abnormal present?
urinalysis results include a dark yellow color, positive d. Why are they not present in the microscopic
bilirubin, and needle-shaped crystals seen on micro- constituents?
scopic examination. e. Based on the family history, what genetic disorder
a. What is the most probable cause of the infant’s should be considered?
jaundice?
4. An 8–month-old boy is admitted to the pediatric unit
b. Could these same urine findings be associated with an with a general diagnosis of failure to thrive. The parents
adult? Explain your answer. have observed slowness in the infant’s development of
c. What kinds of crystals are present? Name another type motor skills. They also mention the occasional appear-
of crystal with a spherical shape that is associated with ance of a substance resembling orange sand in the child’s
this condition. diapers. Urinalysis results are as follows:
d. When blood is drawn from this infant, what precau- COLOR: Yellow KETONES: Negative
tion should be taken to ensure the integrity of the APPEARANCE: Slightly BLOOD: Negative hazy
specimen? SP. GRAVITY: 1.024 BILIRUBIN: Negative
2. A newborn develops severe vomiting and symptoms pH: 5.0 UROBILINOGEN: Normal
of metabolic acidosis. Urinalysis results are positive for PROTEIN: Negative NITRITE: Negative
ketones and negative for glucose and other reducing
GLUCOSE: Negative LEUKOCYTE: Negative
substances.
Microscopic:
a. If the urine had an odor of “sweaty feet,” what meta-
bolic disorder would be suspected? Many uric acid crystals
b. If the newborn was producing dark brown urine with a. Is the urine pH consistent with the appearance of uric
a sweet odor, what disorder would be suspected? acid crystals?
c. Would an MS/MS screen be helpful for the diagnosis? b. Is there any correlation between the urinalysis results
and the substance observed in the child’s diapers?
3. A 13-year-old boy is awakened with severe back and Explain your answer.
abdominal pain and is taken to the emergency depart- c. What disorder do the patient’s history and the urinaly-
ment by his parents. A complete blood count is normal. sis results indicate?
Family history shows that both his father and uncle are
d. Is the fact that this is a male patient of any significance?
chronic kidney stone formers. Results of a urinalysis
Explain your answer.
are as follows:
e. Name the enzyme that is missing.
COLOR: Yellow KETONES: Negative
APPEARANCE: Hazy BLOOD: Moderate 5. Shortly after arriving for the day shift in the urinalysis
laboratory, a technician notices that an undiscarded urine
SP. GRAVITY: 1.025 BILIRUBIN: Negative
has a black color. The previously completed
pH: 6.0 UROBILINOGEN: Normal report indicates the color to be yellow.
PROTEIN: Negative NITRITE: Negative a. Is this observation significant? Explain your answer.
GLUCOSE: Negative LEUKOCYTE: Negative b. The original urinalysis report showed the specimen
Microscopic: to be positive for ketones. Is this significant? Why or
why not?
>15–20 RBCs/hpf Few squamous epithelial
cells c. If the ketones are negative and the pH is 8.0 is this
significant? Why or why not?
0–3 WBCs/hpf Many cystine crystals
a. What condition does the patient’s symptoms represent? 6. Bobby Williams, age 8, is admitted through the emer-
gency department with a ruptured appendix. Although
b. What is the physiologic abnormality causing this
surgery is successful, Bobby’s recovery is slow, and the
condition?
3920_Ch08_163-179 23/01/14 10:24 AM Page 178

178 Part Two | Urinalysis

physicians are concerned about his health prior to the 7. An anemic patient is suspected of having lead poisoning.
ruptured appendix. Bobby’s mother states that he has a. What historical urine test was requested?
always been noticeably underweight despite eating a
b. What should be added to the urine before testing?
balanced diet and having strong appetite and that his
younger brother exhibits similar characteristics. A note c. What element of heme synthesis would this be
in his chart from the first postoperative day reports that testing for?
the evening nurse noticed a blue coloration in the urinary d. Name another substance that can be tested for lead
catheter bag. poisoning.
a. Is the catheter bag color significant? e. What element of heme synthesis would this test for?
b. What condition can be suspected from this history?
c. What is Bobby’s prognosis?
3920_Ch09_180-202 23/01/14 10:30 AM Page 198

198 Part Three | Other Body Fluids

Log on to 16. Whitaker, JN: Myelin basic protein in cerebrospinal fluid and
www.fadavis.com/strasinger other body fluids. Multiple Sclerosis 4(1):16–21, 1998.
for additional content related 17. Okta, M, et al: Evaluation of an enzyme immunoassay for
to this chapter. myelin basic protein in CSF. Clin Chem 46:1326–1330, 2000.
18. Menkes, J: The causes of low spinal fluid sugar in bacterial
meningitis: Another look. Pediatrics 44(1):1–3, 1969.
References 19. Hourani, BT, Hamlin, EM, and Reynolds, TB: Cerebrospinal
fluid glutamine as a measure of hepatic encephalopathy. Arch
1. Scanlon,VC and Sanders,T: Essentials of Anatomy and Physiol- Intern Med 127:1033–1036, 1971.
ogy, 5th ed. FA Davis Company, Philadelphia, 2007. 20. Glasgow, AM, and Dhiensiri, K: Improved assay for spinal fluid
2. Edlow, JA, and Caplan, LR: Avoiding pitfalls in the diagnosis of glutamine and values for children with Reye’s syndrome. Clin
subarachnoid hemorrhage. N Engl J Med 342:29–36, 2000. Chem 20(6):642–644, 1974.
3. Nagda, KK: Procoagulant activity of cerebrospinal fluid in 21. Murray, PR, and Hampton, CM: Recovery of pathogenic bac-
health and disease. Indian J Med Res 74:107–110, 1981. teria from cerebrospinal fluid. J Clin Microbiol 12:554–557,
4. Chow, G, and Schmidley, JW: Lysis of erythrocytes and leukocytes 1980.
in traumatic lumbar punctures. Arch Neurol 41:1084–1085, 1984. 22. Sato, Y, et al: Rapid diagnosis of cryptococcal meningitis by
5. Seehusen, DA, Reeves, MM, and Fomin, DA: Cerebrospinal microscopic examination of centrifuged cerebrospinal fluid
fluid analysis. Am Fam Physician 68(6):1103–1108, 2003. sediment. J Neurol Sci 164(1):72–75, 1999.
6. Glasser, L: Tapping the wealth of information in CSF. Diagn 23. Eng, RHK, and Person, A: Serum cryptococcal antigen determi-
Med 4(1):23–33, 1981. nation in the presence of rheumatoid factor. J Clin Microbiol
7. University of Virginia Health Sciences Center: Clinical Labora- 14:700–702, 1981.
tory Procedure Manual. Charlottesville, VA, 1993. 24. Stockman, L, and Roberts, GD: Specificity of the latex test for
8. Kjeldsberg, CR, and Knight, JA: Body Fluids: Laboratory Exam- cryptococcal antigen: A rapid simple method for eliminating
ination of Amniotic, Cerebrospinal, Seminal, Serous and Syn- interference. J Clin Microbiol 17(5):945–947, 1983.
ovial Fluids: A Textbook Atlas. ASCP, Chicago, 1993. 25. Knight, FR: New enzyme immunoassay for detecting crypto-
9. Abrams, J, and Schumacher, HR: Bone marrow in cerebrospinal coccal antigen. J Clin Pathol 45(9):836–837, 1992.
fluid and possible confusion with malignancy. Arch Pathol Lab 26. Klausner, JD, Vijayan, T, Chiller,T: Sensitivity and specificity of
Med 110:366–369, 1986. a new cryptococcal antigen lateral flow assay in serum and
10. Bentz, JS: Laboratory investigation of multiple sclerosis. Lab cerebrospinal fluid. MLO, March 2013.
Med 26(6):393–399, 1995. 27. Wojewoda,C: Bacterial Antigen Testing, The Good, the Not So
11. Biou, D, et al: Cerebrospinal fluid protein concentrations in Bad and the Ugly. NewsPath. Accessed February 14, 2013
children: Age-related values in patients without disorders of the 28. Leventhal, R and Cheadle, RF: Medical Parasitology, ed.6,
central nervous system. Clin Chem 46(3):399–403, 2000. FA Davis Company, Philadelphia, 2012.
12. Fishman, RA: Cerebrospinal Fluid in Diseases of the Nervous 29. Davis, LE, and Schmitt, JW: Clinical significance of cere-
System, ed 2. WB Saunders, Philadelphia, 1992. brospinal fluid tests for neurosyphilis. Ann Neurol 25:50–53,
13. Hershey, LA, and Trotter, JL: The use and abuse of the cere- 1989.
brospinal fluid IgG profile in the adult: A practical evaluation. 30. Albright, RE, et al: Issues in cerebrospinal fluid management.
Ann Neurol 8(4):426–434, 1980. Am J Clin Pathol 95(3):397–401, 1991.
14. Grimaldi, LME, et al: Oligoclonal IgG bands in cerebrospinal 31. Lofsness, KG, and Jensen, TL: The preparation of simulated
fluid and serum during asymptomatic human immunodefi- spinal fluid for teaching purposes. Am J Med Technology
ciency virus infection. Ann Neurol 24:277–279, 1988. 49(7):493–496, 1983.
15. Rouah, E, Rogers, BB, and Buffone, GJ: Transferrin analysis by
immunofixation as an aid in the diagnosis of cerebrospinal fluid
otorrhea. Arch Pathol Lab Med 111:756–757, 1987.

Study Questions
1. The functions of the CSF include all of the following 3. Substances present in the CSF are controlled by the:
except: A. Arachnoid granulations
A. Removing metabolic wastes B. Blood–brain barrier
B. Producing an ultrafiltrate of plasma C. Presence of one-way valves
C. Supplying nutrients to the CNS D. Blood–CSF barrier
D. Protecting the brain and spinal cord 4. What department is the CSF tube labeled 3 routinely
sent to?
2. The CSF flows through the:
A. Hematology
A. Choroid plexus
B. Chemistry
B. Pia mater C. Microbiology
C. Arachnoid space D. Serology
D. Dura mater
3920_Ch09_180-202 23/01/14 10:30 AM Page 199

Chapter 9 | Cerebrospinal Fluid 199

5. The CSF tube that should be kept at room temperature is: 12. The purpose of adding albumin to CSF before cytocen-
A. Tube 1 trifugation is to:
B. Tube 2 A. Increase the cell yield
C. Tube 3 B. Decrease the cellular distortion
D. Tube 4 C. Improve the cellular staining
D. Both A and B
6. Place the appropriate letter in front of the statement that
best describes CSF specimens in these two conditions: 13. The primary concern when pleocytosis of neutrophils and
A. Traumatic tap lymphocytes is found in the CSF is:
B. Intracranial hemorrhage A. Meningitis
____ Even distribution of blood in all tubes B. CNS malignancy
____ Xanthochromic supernatant C. Multiple sclerosis
____ Concentration of blood in tube 1 is greater D. Hemorrhage
than in tube 3 14. Neutrophils with pyknotic nuclei may be mistaken for:
____ Specimen contains clots A. Lymphocytes
7. The presence of xanthochromia can be caused by all of B. Nucleated RBCs
the following except: C. Malignant cells
A. Immature liver function D. Spindle-shaped cells
B. RBC degradation
15. The presence of which of the following cells is increased
C. A recent hemorrhage in a parasitic infection?
D. Elevated CSF protein A. Neutrophils
8. A web-like pellicle in a refrigerated CSF specimen B. Macrophages
indicates: C. Eosinophils
A. Tubercular meningitis D. Lymphocytes
B. Multiple sclerosis
16. Macrophages appear in the CSF after:
C. Primary CNS malignancy
A. Hemorrhage
D. Viral meningitis
B. Repeated spinal taps
9. Given the following information, calculate the CSF WBC C. Diagnostic procedures
count: cells counted, 80; dilution, 1:10; large Neubauer
D. All of the above
squares counted, 10.
A. 8 17. Nucleated RBCs are seen in the CSF as a result of:
B. 80 A. Elevated blood RBCs
C. 800 B. Treatment of anemia
D. 8000 C. Severe hemorrhage
D. Bone marrow contamination
10. A CSF WBC count is diluted with:
A. Distilled water 18. After a CNS diagnostic procedure, which of the following
might be seen in the CSF?
B. Normal saline
A. Choroidal cells
C. Acetic acid
B. Ependymal cells
D. Hypotonic saline
C. Spindle-shaped cells
11. A total CSF cell count on a clear fluid should be:
D. All of the above
A. Reported as normal
B. Not reported
C. Diluted with normal saline
D. Counted undiluted
3920_Ch09_180-202 23/01/14 10:30 AM Page 200

200 Part Three | Other Body Fluids

19. Hemosiderin granules and hematoidin crystals are seen in: 27. Given the following results, calculate the IgG index: CSF IgG,
A. Lymphocytes 50 mg/dL; serum IgG, 2 g/dL; CSF albumin, 70 mg/dL;
serum albumin, 5 g/dL.
B. Macrophages
A. 0.6
C. Ependymal cells
B. 6.0
D. Neutrophils
C. 1.8
20. Myeloblasts are seen in the CSF:
D. 2.8
A. In bacterial infections
28. The CSF IgG index calculated in Study Question 27
B. In conjunction with CNS malignancy
indicates:
C. After cerebral hemorrhage
A. Synthesis of IgG in the CNS
D. As a complication of acute leukemia
B. Damage to the blood–brain barrier
21. Cells resembling large and small lymphocytes with C. Cerebral hemorrhage
cleaved nuclei represent:
D. Lymphoma infiltration
A. Lymphoma cells
29. The finding of oligoclonal bands in the CSF and not in
B. Choroid cells
the serum is seen with:
C. Melanoma cells
A. Multiple myeloma
D. Medulloblastoma cells
B. CNS malignancy
22. The reference range for CSF protein is: C. Multiple sclerosis
A. 6 to 8 g/dL D. Viral infections
B. 15 to 45 g/dL
30. A CSF glucose of 15 mg/dL, WBC count of 5000,
C. 6 to 8 mg/dL 90% neutrophils, and protein of 80 mg/dL suggests:
D. 15 to 45 mg/dL A. Fungal meningitis
23. CSF can be differentiated from serum by the presence of: B. Viral meningitis
A. Albumin C. Tubercular meningitis
B. Globulin D. Bacterial meningitis
C. Prealbumin 31. A patient with a blood glucose of 120 mg/dL would have
D. Tau transferrin a normal CSF glucose of:
24. In serum, the second most prevalent protein is IgG; in A. 20 mg/dL
CSF, the second most prevalent protein is: B. 60 mg/dL
A. Transferrin C. 80 mg/dL
B. Prealbumin D. 120 mg/dL
C. IgA 32. CSF lactate will be more consistently decreased in:
D. Ceruloplasmin A. Bacterial meningitis
25. Elevated CSF protein values can be caused by all of the B. Viral meningitis
following except: C. Fungal meningitis
A. Meningitis D. Tubercular meningitis
B. Multiple sclerosis
33. Measurement of which of the following can be replaced by
C. Fluid leakage CSF glutamine analysis in children with Reye syndrome?
D. CNS malignancy A. Ammonia
26. The integrity of the blood–brain barrier is measured using B. Lactate
the: C. Glucose
A. CSF/serum albumin index D. α -Ketoglutarate
B. CSF/serum globulin ratio
C. CSF albumin index
D. CSF IgG index
3920_Ch09_180-202 23/01/14 10:30 AM Page 201

Chapter 9 | Cerebrospinal Fluid 201

34. Before performing a Gram stain on CSF, the specimen 36. The test of choice to detect neurosyphilis is the:
must be: A. RPR
A. Filtered B. VDRL
B. Warmed to 37°C C. FAB
C. Centrifuged D. FTA-ABS
D. Mixed
35. All of the following statements are true about cryptococcal
meningitis except:
A. An India ink preparation is positive
B. A starburst pattern is seen on Gram stain
C. The WBC count is over 2000
D. A confirmatory immunology test is available

Case Studies and Clinical Situations


1. Three tubes of CSF containing evenly distributed visible d. What could cause a false-positive reaction in this test?
blood are drawn from a 75-year-old disoriented patient e. If the tests named in a and c are negative, the glucose
and delivered to the laboratory. Initial test results are as level is 35 mg/dL, and a pellicle is observed in the
follows: fluid, what additional testing should be performed?
WBC count: 250 µ L Protein: 150 mg/dL f. If CSF and serum IEF was performed on this patient,
Glucose: 70 mg/dL Gram stain: No organisms what unusual findings might be present?
seen
3. A 35-year-old woman is admitted to the hospital with
Differential: Neutrophils, 68%; monocytes, 3%; lympho- symptoms of intermittent blurred vision, weakness, and
cytes, 28%; eosinophils, 1% loss of sensation in her legs. A lumbar puncture is per-
Many macrophages containing ingested RBCs formed with the following results:
a. What is the most probable condition indicated by Appearance: Colorless, clear
these results? State two reasons for your answer. WBC count: 35 cells/µ L (90% lymphocytes)
b. Are the elevated WBC count and protein significant? Glucose: 60 mg/dL (plasma: 100 mg/dL)
Explain your answer. Protein: 60 mg/dL (serum: 8 g/dL)
c. Are the percentages of the cells in the differential of Albumin: 40 mg/dL (serum: 6 g/dL)
any significance? Explain your answer.
IgG globulin: 20 mg/dL (serum: 2 g/dL)
d. What two other structures besides RBCs might be
contained in the macrophages? a. Name and perform the calculation to determine the
integrity of the patient’s blood–brain barrier.
e. If the blood was unevenly distributed and nucleated
RBCs and capillary structures were seen instead of b. Does the patient have an intact barrier?
macrophages, what would this indicate? c. Name and perform the calculation used to determine
if IgG is being synthesized within the CNS.
2. A patient with AIDS is hospitalized with symptoms of
high fever and rigidity of the neck. Routine laboratory d. What does this result indicate?
tests on the CSF show a WBC count of 100/µ L with a e. Considering the patient’s clinical symptoms and the
predominance of lymphocytes and monocytes, glucose calculation results, what diagnosis is suggested?
of 55 mg/dL (plasma: 85 mg/dL), and a protein of f. If immunofixation electrophoresis is performed on
70 mg/dL. The Gram stain shows a questionable starburst the patient’s serum and CSF, what findings would be
pattern. expected?
a. What additional microscopic examination should be g. What substance in the CSF can be measured to moni-
performed? tor this patient?
b. If the test is positive, what is the patient’s diagnosis?
c. If the results of the test are questionable, what addi-
tional testing can be performed?
3920_Ch09_180-202 23/01/14 10:30 AM Page 202

202 Part Three | Other Body Fluids

4. Mary Howard, age 5, is admitted to the pediatrics ward c. Are the lymphocytes significant? Why or why not?
with a temperature of 105%F, lethargy, and cervical rigid- d. Would a CSF lactate test be of any value for the diag-
ity. A lumbar spinal tap is performed, and three tubes of nosis? Why or why not?
cloudy CSF are delivered to the laboratory. Preliminary
test results are as follows: 5. State possible technical errors that could result in the fol-
lowing discrepancies:
Appearance: Cloudy
a. An unusual number of Gram stains reported as gram-
WBC count: 800 cells/µ L
positive cocci fail to be confirmed by positive cultures.
Differential: 80% lymphocytes, 15% monocytes,
b. A physician complains that CSF differentials are being
5% neutrophils
reported only as polynuclear and mononuclear cells.
Protein: 65 mg/dL
c. Bacteria observed on the cytospin differential cannot
Glucose: 70 mg/dL be confirmed by Gram stain or culture.
Gram stain: No organisms seen d. The majority of CSF specimens sent to the laboratory
a. From these results, what preliminary diagnosis could from the neurology clinic have glucose readings less
the physician consider? than 50% of the corresponding blood glucose results
b. Is the Gram stain result of particular significance? performed in the clinic.
Why or why not?
3920_Ch10_203-216 23/01/14 10:32 AM Page 214

214 Part Three | Other Body Fluids

Study Questions

1. Maturation of spermatozoa takes place in the: 8. An increased semen pH may be caused by:
A. Sertoli cells A. Prostatic infection
B. Seminiferous tubules B. Decreased prostatic secretions
C. Epididymis C. Decreased bulbourethral gland secretions
D. Seminal vesicles D. All of the above

2. Enzymes for the coagulation and liquefaction of semen are 9. Proteolytic enzymes may be added to semen specimens to:
produced by the: A. Increase the viscosity
A. Seminal vesicles B. Dilute the specimen
B. Bulbourethral glands C. Decrease the viscosity
C. Ductus deferens D. Neutralize the specimen
D. Prostate gland 10. The normal sperm concentration is:
3. The major component of seminal fluid is: A. Less than 20 million/µ L
A. Glucose B. More than 20 million/mL
B. Fructose C. Less than 20 million/mL
C. Acid phosphatase D. More than 20 million/µ L
D. Citric acid 11. Given the following information, calculate the sperm con-
centration: dilution, 1:20; sperm counted in five RBC
4. If the first portion of a semen specimen is not collected, the squares on each side of the hemocytometer, 80 and 86;
semen analysis will have which of the following? volume, 3 mL.
A. Decreased pH A. 80 million/mL
B. Increased viscosity B. 83 million/mL
C. Decreased sperm count C. 86 million/mL
D. Decreased sperm motility D. 169 million/µ L
5. Failure of laboratory personnel to document the time a 12. Using the above information, calculate the sperm concen-
semen sample is collected primarily affects the interpreta- tration when 80 sperm are counted in 1 WBC square and
tion of semen: 86 sperm are counted in another WBC square.
A. Appearance A. 83 million/mL
B. Volume B. 166 million per ejaculate
C. pH C. 16.6 million/mL
D. Viscosity D. 50 million per ejaculate
6. Liquefaction of a semen specimen should take place within: 13. The primary reason to dilute a semen specimen before
A. 1 hour performing a sperm concentration is to:
B. 2 hours A. Immobilize the sperm
C. 3 hours B. Facilitate the chamber count
D. 4 hours C. Decrease the viscosity
D. Stain the sperm
7. A semen specimen delivered to the laboratory in a condom
has a normal sperm count and markedly decreased sperm
motility. This indicates:
A. Decreased fructose
B. Antispermicide in the condom
C. Increased semen viscosity
D. Increased semen alkalinity
3920_Ch10_203-216 23/01/14 10:32 AM Page 215

Chapter 10 | Semen 215

14. When performing a sperm concentration, 60 sperm are 22. Additional parameters measured by Kruger’s strict mor-
counted in the RBC squares on one side of the hemocy- phology include all of the following except:
tometer and 90 sperm are counted in the RBC squares on A. Vitality
the other side. The specimen is diluted 1:20. The:
B. Presence of vacuoles
A. Specimen should be rediluted and counted
C. Acrosome size
B. Sperm count is 75 million/mL
D. Tail length
C. Sperm count is greater than 5 million/mL
23. Round cells that are of concern and may be included in
D. Sperm concentration is abnormal
sperm counts and morphology analysis are:
15. Sperm motility evaluations are performed: A. Leukocytes
A. Immediately after the specimen is collected B. Spermatids
B. Within 1 hour of collection C. RBCs
C. After 3 hours of incubation D. Both A and B
D. At 6-hour intervals for 1 day
24. If 5 round cells per 100 sperm are counted in a sperm mor-
16. The percentage of sperm showing average motility that is phology smear and the sperm concentration is 30 million,
considered normal is: the concentration of round cells is:
A. 25% A. 150,000
B. 50% B. 1.5 million
C. 60% C. 300,000
D. 75% D. 15 million
17. The purpose of the acrosomal cap is to: 25. Following an abnormal sperm motility test with a normal
A. Penetrate the ovum sperm count, what additional test might be ordered?
B. Protect the the nucleus A. Fructose level
C. Create energy for tail movement B. Zinc level
D. Protect the neckpiece C. MAR test
D. Eosin-nigrosin stain
18. The sperm part containing a mitochondrial sheath is the:
A. Head 26. Follow-up testing for a low sperm concentration would
include testing for:
B. Neckpiece
A. Antisperm antibodies
C. Midpiece
B. Seminal fluid fructose
D. Tail
C. Sperm vitality
19. All of the following are associated with sperm motility
D. Prostatic acid phosphatase
except the:
A. Head 27. The immunobead test for antisperm antibodies:
B. Neckpiece A. Detects the presence of male antibodies
C. Midpiece B. Determines the presence of IgG, IgM, and IgA
antibodies
D. Tail
C. Determines the location of antisperm antibodies
20. The morphologic shape of a normal sperm head is:
D. All of the above
A. Round
28. Measurement of α -glucosidase is performed to detect a
B. Tapered
disorder of the:
C. Oval
A. Seminiferous tubules
D. Amorphous
B. Epididymis
21. Normal sperm morphology when using the WHO criteria is: C. Prostate gland
A. >30% normal forms D. Bulbourethral glands
B. <30% normal forms
C. >15% abnormal forms
D. <15% normal forms
3920_Ch10_203-216 23/01/14 10:32 AM Page 216

216 Part Three | Other Body Fluids

29. A specimen delivered to the laboratory with a request 31. Standardization of procedures and reference values for
for prostatic acid phosphatase and glycoprotein p30 was semen analysis is primarily provided by the:
collected to determine: A. Manufacturers of instrumentation
A. Prostatic infection B. WHO
B. Presence of antisperm antibodies C. Manufacturers of control samples
C. A possible rape D. Clinical Laboratory Improvement Amendments
D. Successful vasectomy
30. Following a negative postvasectomy wet preparation, the
specimen should be:
A. Centrifuged and reexamined
B. Stained and reexamined
C. Reported as no sperm seen
D. Both A and B

Case Studies and Clinical Situations


1. A repeat semen analysis for fertility testing is reported as d. State three ways in which a positive result on these
follows: tests could be affecting male fertility.
VOLUME: 3.5 mL SPERM COUNT: 6 million/mL 3. A yellow-colored semen specimen is received in the labo-
VISCOSITY: Normal SPERM MOTILITY: 30%— ratory. The analysis is normal except for decreased sperm
grade 1.0 motility. Explain the possible connection between the two
pH: 7.5 MORPHOLOGY: <30% normal abnormal findings.
forms—30 spermatids/100 sperm 4. Abnormal results of a semen analysis are volume = 1.0 mL
The results correspond with the first analysis. and sperm concentration = 1 million/mL. State a non-
a. List three abnormal parameters. pathologic cause of these abnormal results.
b. What is the sperm concentration? Is this normal? 5. A semen specimen with normal initial appearance fails to
c. What is the spermatid count? Is this normal? liquefy after 60 minutes.
d. Could the sperm concentration and the spermatid a. Would a specimen pH of 9.0 be consistent with this
count be related to the infertility? Explain your answer. observation? Why or why not?
b. State three chemical tests that would be of value in
2. A semen analysis on a vasovasostomy patient has a nor-
this analysis.
mal sperm concentration; however, motility is decreased,
and clumping is observed on the wet preparation. c. How does this abnormality affect fertility?
a. Explain the possible connection between these obser- 6. A specimen is delivered to the laboratory with a request to
vations and the patient’s recent surgery. determine if semen is present.
b. What tests could be performed to further evaluate the a. What two chemical tests could be performed on the
patient’s infertility? specimen?
c. Briefly explain the different interpretations offered by b. What additional examination could be performed on
these two tests. the specimen?
3920_Ch11_217-228 23/01/14 10:34 AM Page 225

Chapter 11 | Synovial Fluid 225

References 4. Brown, W, et al: Validation of body fluid analysis on the Coulter


LH 750. Lab Hem 9(3):155–159, 2004.
1. Shmerling, RH: Synovial fluid analysis. A critical reappraisal.
5. Schumacher, HD, Clayburne, G, and Chen, L: Synovial fluid
Rheum Dis Clin North Am 20(2):503–512, 1994.
aspiration and analysis in evaluation of gout and other crystal-
2. Clinical and Laboratory Science Institute (CLSI): Analysis of
induced diseases. Arthritis Foundation: Bulletin on the
Body Fluids in Clinical Chemistry, Approved Guideline, C49-A,
Rheumatic Diseases 53(3), 2004.
Wayne, PA, 2007, CLSI.
6. Harris, MD, Siegel, LB, and Alloway, JA: Gout and hyperuricemia.
3. Smith, GP, and Kjeldsberg, CR: Cerebrospinal, synovial, and serous
Am Fam Physician 59(4):925–934, 1999.
body fluids. In Henry, JB (ed): Clinical Diagnosis and Management
7. Cornbleet, PJ: Synovial fluid crystal analysis. Lab Med 28(12):
by Laboratory Methods. WB Saunders, Philadelphia, 2001.
774–779, 1997.

Study Questions
1. The functions of synovial fluid include all of the following 6. Normal synovial fluid resembles:
except: A. Egg white
A. Lubrication for the joints B. Normal serum
B. Removal of cartilage debris C. Dilute urine
C. Cushioning joints during jogging D. Lipemic serum
D. Providing nutrients for cartilage 7. Before testing, very viscous synovial fluid should be
2. The primary function of synoviocytes is to: treated with:
A. Provide nutrients for the joints A. Normal saline
B. Secrete hyaluronic acid B. Hyaluronidase
C. Regulate glucose filtration C. Distilled water
D. Prevent crystal formation D. Hypotonic saline

3. Which of the following is not a frequently performed test 8. Addition of a cloudy, yellow synovial fluid to acetic acid
on synovial fluid? produces a/an:
A. Uric acid A. Yellow-white precipitate
B. WBC count B. Easily dispersed clot
C. Crystal examination C. Solid clot
D. Opalescent appearance
D. Gram stain
9. Which of the following could be the most significantly
4. The procedure for collecting synovial fluid is called:
affected if a synovial fluid is refrigerated before testing?
A. Synovialcentesis
A. Glucose
B. Arthrocentesis
B. Crystal examination
C. Joint puncture
C. Mucin clot test
D. Arteriocentesis
D. Differential
5. Match the following disorders with their appropriate group: 10. The highest WBC count can be expected to be seen with:
A. Noninflammatory A. Noninflammatory arthritis
B. Inflammatory B. Inflammatory arthritis
C. Septic C. Septic arthritis
D. Hemorrhagic D. Hemorrhagic arthritis
____ Gout
11. When diluting a synovial fluid WBC count, all of the
____ Neisseria gonorrhoeae infection following are acceptable except:
____ Systemic lupus erythematosus A. Acetic acid
____ Osteoarthritis B. Isotonic saline
____ Hemophilia C. Hypotonic saline
____ Rheumatoid arthritis D. Saline with saponin
____ Heparin overdose
3920_Ch11_217-228 23/01/14 10:34 AM Page 226

226 Part Three | Other Body Fluids

12. The lowest percentage of neutrophils would be seen in: 19. In an examination of synovial fluid under compensated
A. Noninflammatory arthritis polarized light, rhomboid-shaped crystals are observed.
What color would these crystals be when aligned parallel
B. Inflammatory arthritis
to the slow vibration?
C. Septic arthritis
A. White
D. Hemorrhagic arthritis
B. Yellow
13. All of the following are abnormal when seen in synovial C. Blue
fluid except:
D. Red
A. Neutrophages
20. If crystals shaped like needles are aligned perpendicular
B. Ragocytes
to the slow vibration of compensated polarized light, what
C. Synovial lining cells color are they?
D. Lipid droplets A. White
14. Synovial fluid crystals that occur as a result of purine B. Yellow
metabolism or chemotherapy for leukemia are: C. Blue
A. Monosodium urate D. Red
B. Cholesterol
21. Negative birefringence occurs under red-compensated po-
C. Calcium pyrophosphate larized light when:
D. Apatite A. Slow light is impeded more than fast light
15. Synovial fluid crystals associated with inflammation in B. Slow light is less impeded than fast light
dialysis patients are: C. Fast light runs against the molecular grain of the
A. Calcium pyrophosphate dihydrate crystal
B. Calcium oxalate D. Both B and C
C. Corticosteroid 22. Synovial fluid cultures are often plated on chocolate agar
D. Monosodium urate to detect the presence of:
16. Crystals associated with pseudogout are: A. Neisseria gonorrhoeae
A. Monosodium urate B. Staphylococcus agalactiae
B. Calcium pyrophosphate dihydrate C. Streptococcus viridans
C. Apatite D. Enterococcus faecalis
D. Corticosteroid 23. The most frequently performed chemical test on synovial
fluid is:
17. Synovial fluid for crystal examination should be examined
as a/an: A. Total protein
A. Wet preparation B. Uric acid
B. Wright's stain C. Calcium
C. Gram stain D. Glucose
D. Acid-fast stain 24. Which of the following chemistry tests can be performed
on synovial fluid to determine the severity of RA?
18. Crystals that have the ability to polarize light are:
A. Glucose
A. Corticosteroid
B. Protein
B. Monosodium urate
C. Lactate
C. Calcium oxalate
D. Uric acid
D. All of the above
25. Serologic tests on patients’ serum may be performed to
detect antibodies causing arthritis for all of the following
disorders except:
A. Pseudogout
B. Rheumatoid arthritis
C. Systemic lupus erythematosus
D. Lyme arthritis
3920_Ch11_217-228 23/01/14 10:34 AM Page 227

Chapter 11 | Synovial Fluid 227

Case Studies and Clinical Situations


1. A 50-year-old man presents in the emergency department d. State two additional tests that could be run to
with severe pain and swelling in the right knee. Arthro- determine the classification.
centesis is performed and 20 mL of milky synovial fluid is
3. Fluid obtained from the knee of an obese 65-year-old
collected. The physician orders a Gram stain, culture, and woman being evaluated for a possible knee replacement
crystal examination of the fluid, as well as a serum uric has the following results:
acid. She requests that the synovial fluid be saved for
APPEARANCE: Pale yellow and hazy
possible additional tests.
WBC COUNT: 500 cells/␮L
a. Describe the tubes into which the fluid would be
routinely placed. GRAM STAIN: Negative
b. If the patient’s serum uric acid level is elevated, what GLUCOSE: 110 mg/dL (serum glucose: 115 mg/dL)
type of crystals and disorder are probable? a. What classification of joint disorder do these results
c. Describe the appearance of these crystals under direct suggest?
and compensated polarized light. b. Under electron microscopy, what crystals might be
detected?
d. Why were the Gram stain and culture ordered?
c. How does the glucose result aid in the disorder
2. A medical laboratory science student dilutes a synovial classification?
fluid sample before performing a WBC count. The fluid
forms a clot. 4. A synovial fluid sample delivered to the laboratory for a
cell count is clotted.
a. Why did the clot form?
a. What abnormal constituent is present in the fluid?
b. How can the student perform a correct dilution of the
fluid? b. What type of tube should be sent to the laboratory for
a cell count?
c. After the correct dilution is made, the WBC count is
c. Could the original tube be used for a Gram stain and
100,000/␮L. State two arthritis classifications that
culture? Why or why not?
could be considered.
3920_Ch12_229-242 23/01/14 10:35 AM Page 240

240 Part Three | Other Body Fluids

Serologic Tests 4. Kjeldsberg, CR, and Knight, JA: Body Fluids: Laboratory
Examination of Amniotic, Cerebrospinal, Seminal, Serous and
Measurement of the tumor markers CEA and CA 125 is a valu- Synovial Fluids. A Textbook Atlas. ASCP, Chicago, 1993.
able procedure for identifying the primary source of tumors 5. Valdez, L, et al: Cholesterol: A useful parameter for distinguish-
producing ascitic exudates. The presence of CA 125 antigen ing between pleural exudates and transudates. Chest 99(5):
1097–1102, 1991.
with a negative CEA suggests the source is from the ovaries, 6. Porcel, JM, and Light, RW: Diagnostic approach to pleural fluid
fallopian tubes, or endometrium.7 effusion in adults. Am Fam Physician 73:1211–1220, 2006.
7. Colice, GL, et al: Medical and surgical treatment of parapneu-
monic effusions: An evidence-based guideline. Chest
Log on to 118:1158–1171, 2000.
www.fadavis.com/strasinger 8. Porcel, JM, et al: Use of a panel of tumor markers (carcinoem-
for additional content related bryonic antigen, cancer antigen 125, carbohydrate antigen 15-3
to this chapter. and cytokeratin 19 fragments) in pleural fluid for differential
diagnosis of benign and malignant effusions. Chest 126:
References 1757–1763, 2004.
1. Clinical and Laboratory Standards Institute (CLSI): Body Fluid 9. Runyan, BA, et al: The serum-ascites albumin gradient is
Analysis for Cellular Composition, Approved Guideline C49-A, superior to the exudate transudate concept in the differential
Wayne, PA, 2006. diagnosis of ascites. Ann Intern Med 117:215–218, 1992.
2. Jay, SJ: Pleural effusions: Definitive evaluation of the exudate. 10. Clinical and Laboratory Science Institute (CLSI): Analysis of
Postgrad Med 80(5):180–188, 1986. Body Fluids in Clinical Chemistry, Approved Guideline,
3. Light, RW: Clinical practice: Pleural effusion. N Engl J Med C49-A, Wayne, PA, 2007, CLSI.
346:1971–1977, 2002.

Study Questions
1. The primary purpose of serous fluid is to: 5. An increase in the amount of serous fluid is called a/an:
A. Remove waste products A. Exudate
B. Lower capillary pressure B. Transudate
C. Lubricate serous membranes C. Effusion
D. Nourish serous membranes D. Malignancy

2. The membrane that lines the wall of a cavity is the: 6. Pleural fluid is collected by:
A. Visceral A. Pleurocentesis
B. Peritoneal B. Paracentesis
C. Pericentesis
C. Pleural
D. Thoracentesis
D. Parietal
7. Place the appropriate letter in front of the following state-
3. During normal production of serous fluid, the slight excess
ments describing transudates and exudates.
of fluid is:
A. Transudate
A. Absorbed by the lymphatic system
B. Exudate
B. Absorbed through the visceral capillaries
____ Caused by increased hydrostatic pressure
C. Stored in the mesothelial cells
____ Caused by increased capillary permeability
D. Metabolized by the mesothelial cells ____ Caused by decreased oncotic pressure
4. Production of serous fluid is controlled by: ____ Caused by congestive heart failure
A. Capillary oncotic pressure ____ Malignancy related
B. Capillary hydrostatic pressure ____ Tuberculosis related
C. Capillary permeability ____ Endocarditis related
D. All of the above ____ Clear appearance
3920_Ch12_229-242 23/01/14 10:35 AM Page 241

Chapter 12 | Serous Fluid 241

8. Fluid:serum protein and lactic dehydrogenase ratios are 16. A pleural fluid pH of 6.0 indicates:
performed on serous fluids: A. Esophageal rupture
A. When malignancy is suspected B. Mesothelioma
B. To classify transudates and exudates C. Malignancy
C. To determine the type of serous fluid D. Rheumatoid effusion
D. When a traumatic tap has occurred
17. Plasma cells seen in pleural fluid indicate:
9. Which of the following requires the most additional A. Bacterial endocarditis
testing?
B. Primary malignancy
A. Transudate
C. Metastatic lung malignancy
B. Exudate
D. Tuberculosis infection
10. An additional test performed on pleural fluid to classify
18. A significant cell found in pericardial or pleural fluid that
the fluid as a transudate or exudate is the:
should be referred to cytology is a:
A. WBC count
A. Reactive lymphocyte
B. RBC count
B. Mesothelioma cell
C. Fluid:cholesterol ratio
C. Monocyte
D. Fluid-to-serum protein gradient
D. Mesothelial cell
11. A milky-appearing pleural fluid indicates:
19. Another name for a peritoneal effusion is:
A. Thoracic duct leakage
A. Peritonitis
B. Chronic inflammation
B. Lavage
C. Microbial infection
C. Ascites
D. Both A and B
D. Cirrhosis
12. Which of the following best represents a hemothorax?
20. A test performed primarily on peritoneal lavage fluid is
A. Blood HCT: 42 Fluid HCT: 15 a/an:
B. Blood HCT: 42 Fluid HCT: 10 A. WBC count
C. Blood HCT: 30 Fluid HCT: 10 B. RBC count
D. Blood HCT: 30 Fluid HCT: 20 C. Absolute neutrophil count
13. All of the following are normal cells seen in pleural fluid D. Amylase
except:
21. The recommended test for determining whether peri-
A. Mesothelial cells toneal fluid is a transudate or an exudate is the:
B. Neutrophils A. Fluid:serum albumin ratio
C. Lymphocytes B. Serum ascites albumin gradient
D. Mesothelioma cells C. Fluid:serum lactic dehydrogenase ratio
14. A differential observation of pleural fluid associated with D. Absolute neutrophil count
tuberculosis is:
22. Given the following results, classify this peritoneal fluid:
A. Increased neutrophils serum albumin, 2.2 g/dL; serum protein, 6.0 g/dL; fluid
B. Decreased lymphocytes albumin, 1.6 g/dL.
C. Decreased mesothelial cells A. Transudate
D. Increased mesothelial cells B. Exudate
15. All of the following are characteristics of malignant cells 23. Differentiation between bacterial peritonitis and cirrhosis
except: is done by performing a/an:
A. Cytoplasmic molding A. WBC count
B. Absence of nucleoli B. Differential
C. Mucin-containing vacuoles C. Absolute neutrophil count
D. Increased nucleus:cytoplasm ratio D. Absolute lymphocyte count
3920_Ch12_229-242 23/01/14 10:35 AM Page 242

242 Part Three | Other Body Fluids

24. Detection of the CA 125 tumor marker in peritoneal fluid 26. Cultures of peritoneal fluid are incubated:
indicates: A. Aerobically
A. Colon cancer B. Anaerobically
B. Ovarian cancer C. At 37°C and 42°C
C. Gastric malignancy D. Both A and B
D. Prostate cancer
25. Chemical tests primarily performed on peritoneal fluid in-
clude all of the following except:
A. Lactose dehydrogenase
B. Glucose
C. Alkaline phosphatase
D. Amylase

Case Studies and Clinical Situations


1. Fluid from a patient with congestive heart failure is col- 4. Paracentesis is performed on a patient with ascites. The
lected by thoracentesis and sent to the laboratory for fluid appears turbid and has an elevated WBC count.
testing. It appears clear and pale yellow and has a WBC Additional tests ordered include an absolute granulocyte
count of 450/mL, fluid:serum protein ratio of 0.35, and count, amylase, creatinine, CEA, and CA 125.
fluid:serum LD ratio of 0.46. a. What is the purpose for the absolute granulocyte
a. What type of fluid was collected? count? If it is less than 250 cells/mL, what condition
b. Based on the laboratory results, would this fluid be is indicated?
considered a transudate or an exudate? Why? b. If the amylase level is elevated, what is its significance?
c. List two other tests that could be performed to aid in State an additional test that might be ordered.
classifying this fluid. c. Explain the significance of an elevated creatinine level.
2. A cloudy pleural fluid has a glucose level of 30 mg/dL d. What is the purpose of the CEA and CA 125 tests?
(serum glucose level is 100 mg/dL) and a pH of 6.8. 5. Describe a situation in which paracentesis might be
a. What condition do these results indicate? performed on a patient who does not have ascites. If the
b. What additional treatment might the patient receive, RBC count is 300,000/mL, what does this indicate?
based on these results? 6. Microscopic examination of an ascitic fluid shows many
3. The following results were obtained on a peritoneal fluid: cells with nuclear and cytoplasmic irregularities contain-
serum albumin, 2.8 g/dL; fluid albumin, 1.2 g/dL. ing psammoma bodies. The CEA test result is normal.
What additional test would be helpful?
a. Calculate the SAAG.
b. Is this a transudate or an exudate? Why?
c. What is the most probable cause of the effusion?
3920_Ch13_243-254 23/01/14 10:37 AM Page 251

Chapter 13 | Amniotic Fluid 251

4. Wenk, RE, and Rosenbaum, JM: Examination of amniotic fluid. 14. Chapman, JF: Current methods for evaluating FLM. Lab
In Henry, JB (ed): Clinical Diagnosis and Management by Med 17(10):597–602, 1986.
Laboratory Methods. WB Saunders, Philadelphia, 1996. 15. Saad, SA, et al: The reliability and clinical use of a rapid
5. Heron, HJ: The use of the Fern test to differentiate amniotic phosphatidyl glycerol assay in normal and diabetic pregnan-
fluid from urine. Triangle Jul:20:60–62, 1963. cies. Am J Obstet Gynecol 157(6):1516–1520, 1987.
6. Liley, AW: Liquor amnii analysis in the management of the 16. Winn-McMillan, T, and Karon, BS: Comparison of the
pregnancy complicated by Rhesus sensitization. Am J Obstet TDx-FLM II and lecithin to sphingomyelin ratio assays in
Gynecol 82:1359, 1961. predicting fetal lung maturity. Am J Obstet Gynecol
7. Spinnato, JA, et al: Amniotic bilirubin and fetal hemolytic 193,778–782, 2005.
disease. Am J Obstet Gynecol 165(4):1030–1035, 1991. 17. TDx/TDxFLx Fetal Lung Maturity II (FLM II) package insert.
8. McDonald, OL, and Watts, MT: Use of commercially prepared Reference 7A76. Abbott Laboratories, Diagnostics Division,
control sera as quality control materials for spectrophotometric Abbott Park, IL, April 2003.
bilirubin determinations in amniotic fluid. Am J Clin Pathol 18. Clinical and Laboratory Standards Institute. Assessment of fetal
84(4):513–517, 1985. lung maturity by the lamellar body count; approved guideline,
9. Grenache, D: Fetal lung maturity testing: what labs need to CLSI document C58-A. Wayne, PA, CLSI, 2011.
know now. Medical Lab Observer (MLO), February 2012. 19. Khazardoost, S, et al: Amniotic fluid lamellar body count and
10. Gluck, L, et al: Diagnosis of the respiratory distress syndrome its sensitivity and specificity in evaluating of fetal lung maturity.
by amniocentesis. Am J Obstet Gynecol 109(3):440–445, 1971. J Obstet Gynaecol 25(3):257–259, 2005.
11. Dubin, SB: Assessment of FLM: Practice parameter. Am J Clin 20. Sbarra, AJ, et al: Correlation of amniotic fluid optical density at
Pathol 110:723–732, 1998. 650 nm and lecithin/sphingomyelin ratios. Obstet Gynecol
12. Kulovich, MV, Hallman, MB, and Gluck, L: The lung profile: 48:613, 1976.
Normal pregnancy. Am J Obstet Gynecol 135:57–60, 1979. 21. Lu Ji, Gronowski, AM, Eby, C,: Lamellar Body Counts Performed
13. Eisenbrey, AB, et al: Phosphatidyl glycerol in amniotic fluid: on Automated Hematology Analyzers to Assess Fetal Lung Matu-
Comparison of an “ultrasensitive” immunologic assay with TLC rity. LabMedicine 39(7): 419–423, 2008.
and enzymatic assay. Am J Clin Pathol 91(3):293–297, 1989.

Study Questions
1. Which of the following is not a function of amniotic fluid? 5. Amniotic fluid specimens are placed in amber-colored tubes
A. Allows movement of the fetus prior to sending them to the laboratory to prevent the
destruction of:
B. Allows carbon dioxide and oxygen exchange
A. Alpha-fetoprotein
C. Protects fetus from extreme temperature changes
B. Bilirubin
D. Acts as a protective cushion for the fetus
C. Cells for cytogenetics
2. What is the primary cause of the normal increase in amni- D. Lecithin
otic fluid as a pregnancy progresses?
6. How are specimens for FLM testing delivered to and stored
A. Fetal cell metabolism in the laboratory?
B. Fetal swallowing A. Delivered on ice and refrigerated
C. Fetal urine B. Immediately centrifuged
D. Transfer of water across the placenta C. Kept at room temperature
3. Which of the following is not a reason for decreased D. Delivered in a vacuum tube
amounts of amniotic fluid? 7. Why are amniotic specimens for cytogenetic analysis incu-
A. Fetal failure to begin swallowing bated at 37°C prior to analysis?
B. Increased fetal swallowing A. To detect the presence of meconium
C. Membrane leakage B. To differentiate amniotic fluid from urine
D. Urinary tract defects C. To prevent photo-oxidation of bilirubin to biliverdin
4. Why might a creatinine level be requested on an amniotic D. To prolong fetal cell viability and integrity
fluid? 8. Match the following colors in amniotic fluid with their
A. Detect oligohydramnios significance.
B. Detect polyhydramnios ___ A. Colorless 1. Fetal death
C. Differentiate amniotic fluid from maternal urine ___ B. Dark green 2. Normal
D. Evaluate lung maturity
___ C. Red-brown 3. Presence of bilirubin
___ D. Yellow 4. Presence of meconium
3920_Ch13_243-254 23/01/14 10:37 AM Page 252

252 Part Three | Other Body Fluids

9. A significant rise in the OD of amniotic fluid at 450 nm 15. When performing an L/S ratio by thin-layer chromatography,
indicates the presence of which analyte? a mature fetal lung will show:
A. Bilirubin A. Sphingomyelin twice as concentrated as lecithin
B. Lecithin B. No sphingomyelin
C. Oxyhemoglobin C. Lecithin twice as concentrated as sphingomyelin
D. Sphingomyelin D. Equal concentrations of lecithin and sphingomyelin
10. Plotting the amniotic fluid OD on a Liley graph represents 16. True or False: Phosphatidyl glycerol is present with an L/S
the severity of hemolytic disease of the newborn. A value ratio of 1.1.
that is plotted in zone II indicates what condition of the
17. A rapid immunologic test for FLM that does not require
fetus?
performance of thin-layer chromatography is:
A. No hemolysis
A. AFP levels
B. Mildly affected fetus
B. Amniotic acetylcholinesterase
C. Moderately affected fetus that requires close monitoring
C. Aminostat-FLM
D. Severely affected fetus that requires intervention
D. Bilirubin scan
11. The presence of a fetal neural tube disorder may be
18. Does the failure to produce bubbles in the Foam Stability
detected by:
Index indicate increased or decreased lecithin?
A. Increased amniotic fluid bilirubin
A. Increased
B. Increased maternal serum alpha-fetoprotein
B. Decreased
C. Decreased amniotic fluid phosphatidyl glycerol
19. The presence of phosphatidyl glycerol in amniotic fluid
D. Decreased maternal serum acetylcholinesterase
fetal lung maturity tests must be confirmed when:
12. True or False: An AFP MoM value greater than two times A. Hemolytic disease of the newborn is present
the median value is considered an indication of a neural
B. The mother has maternal diabetes
tube disorder.
C. Amniotic fluid is contaminated by hemoglobin
13. When severe HDN is present, which of the following tests
D. Neural tube disorder is suspected
on the amniotic fluid would the physician not order to
determine whether the fetal lungs are mature enough to 20. A lamellar body count of 50,000 correlates with:
withstand a premature delivery? A. Absent phosphatidyl glycerol and L/S ratio of 1.0
A. AFP levels B. L/S ratio of 1.5 and absent phosphatidyl glycerol
B. Foam stability index C. OD at 650 nm of 1.010 and an L/S ratio of 1.1
C. Lecithin/sphingomyelin ratio D. OD at 650 nm of 0.150 and an L/S ratio of 2.0
D. Phosphatidyl glycerol detection
14. True or False: Prior to 35 weeks’ gestation, the normal L/S
ratio is less than 1.6.

Case Studies and Clinical Situations


1. Amniocentesis is performed on a woman believed to be c. What information did this test provide for the physician?
in approximately the 31st week of gestation. This is the d. Why did the physician prefer an Aminostat-FLM to an
second pregnancy for this Rh-negative woman with dia- L/S ratio in this situation?
betes. Spectrophotometric analysis of the fluid shows a
2. Amniocentesis is performed following a maternal serum
∆ A450 of 0.3.
AFP level of 2.2 MoM at 15 weeks’ gestation.
a. Based on the Liley graph, should the physician
a. What fetal condition is suspected?
consider inducing labor?
b. If the amniotic fluid AFP is 2.5 MoM, what additional
b. What else must the physician consider prior to inducing test could be performed?
labor?
c. In what situation would this additional test not be
The physician decides to induce labor based on a performed?
positive Aminostat-FLM.
3920_Ch13_243-254 23/01/14 10:37 AM Page 253

Chapter 13 | Amniotic Fluid 253

3. How might a dark green amniotic fluid affect the results 5. Amniocentesis is performed on a woman whose last two
of the following tests? pregnancies resulted in stillbirths due to hemolytic disease
a. Foam Stability Index of the newborn. A screening test performed at the hospital
is positive for bilirubin, and the specimen is sent to a
b. L/S ratio
reference laboratory for a bilirubin scan. Physicians are
c. Aminostat-FLM concerned when the report comes back negative. What
d. OD650 factors would be considered in evaluating this result.
4. How might a blood-streaked amniotic fluid affect the a. Correct specimen was sent
results of the following tests? b. Specimen was refrigerated
a. L/S ratio c. Specimen was exposed to light
b. AChE d. Specimen reached the reference lab within 30 mins
c. Bilirubin analysis
d. Aminostat-FLM
3920_Ch14_255-268 23/01/14 10:39 AM Page 266

266 Part Three | Other Body Fluids

Study Questions
1. In what part of the digestive tract do pancreatic enzymes 8. Stools from persons with steatorrhea will contain excess
and bile salts contribute to digestion? amounts of:
A. Large intestine A. Barium sulfate
B. Liver B. Blood
C. Small intestine C. Fat
D. Stomach D. Mucus
2. Where does the reabsorption of water take place in the 9. Which of the following pairings of stool appearance and
primary digestive process? cause does not match?
A. Large intestine
A. Black, tarry: blood
B. Pancreas
B. Pale, frothy: steatorrhea
C. Small intestine
C. Yellow-gray: bile duct obstruction
D. Stomach
D. Yellow-green: barium sulfate
3. Which of the following tests is not performed to detect
osmotic diarrhea? 10. Stool specimens that appear ribbon-like are indicative of
which condition?
A. Clinitest
A. Bile-duct obstruction
B. Fecal fats
B. Colitis
C. Fecal neutrophils
D. Muscle fibers C. Intestinal constriction
D. Malignancy
4. The normal composition of feces includes all of the following
except: 11. A black tarry stool is indicative of:
A. Bacteria A. Upper GI bleeding
B. Blood B. Lower GI bleeding
C. Electrolytes C. Excess fat
D. Water D. Excess carbohydrates
5. What is the fecal test that requires a 3-day specimen? 12. Chemical screening tests performed on feces include all
A. Fecal occult blood of the following except:
B. APT test A. APT test
C. Elastase I B. Clinitest
D. Quantitative fecal fat testing C. Pilocarpine iontophoresis
6. The normal brown color of the feces is produced by: D. Quantitative fecal fats
A. Cellulose 13. Secretory diarrhea is caused by:
B. Pancreatic enzymes A. Antibiotic administration
C. Undigested foodstuffs B. Lactose intolerance
D. Urobilin
C. Celiac sprue
7. Diarrhea can result from all of the following except: D. Vibrio cholerae
A. Addition of pathogenic organisms to the normal
14. The fecal osmotic gap is elevated in which disorder?
intestinal flora
B. Disruption of the normal intestinal bacterial flora A. Dumping syndrome

C. Increased concentration of fecal electrolytes B. Osmotic diarrhea


D. Increased reabsorption of intestinal water and C. Secretory diarrhea
electrolytes D. Steatorrhea
3920_Ch14_255-268 23/01/14 10:39 AM Page 267

Chapter 14 | Fecal Analysis 267

15. Microscopic examination of stools provides preliminary 23. What is the recommended number of samples that should
information as to the cause of diarrhea because: be tested to confirm a negative occult blood result?
A. Neutrophils are present in conditions caused by A. One random specimen
toxin-producing bacteria B. Two samples taken from different parts of three
B. Neutrophils are present in conditions that affect the stools
intestinal wall C. Three samples taken from the outermost portion of
C. Red and white blood cells are present if the cause is the stool
bacterial D. Three samples taken from different parts of two
D. Neutrophils are present if the condition is of nonbac- stools
terial etiology
24. The immunochemical tests for occult blood:
16. True or False: The presence of fecal neutrophils would be A. Test for human globulin
expected with diarrhea caused by a rotavirus.
B. Give false-positive reactions with meat hemoglobin
17. Large orange-red droplets seen on direct microscopic C. Can give false-positive reactions with aspirin
examination of stools mixed with Sudan III represent:
D. Are inhibited by porphyrin
A. Cholesterol
25. Guaiac tests for detecting occult blood rely on the:
B. Fatty acids
A. Reaction of hemoglobin with hydrogen peroxide
C. Neutral fats
B. Pseudoperoxidase activity of hemoglobin
D. Soaps
C. Reaction of hemoglobin with ortho-toluidine
18. Microscopic examination of stools mixed with Sudan III
D. Pseudoperoxidase activity of hydrogen peroxide
and glacial acetic acid and then heated will show small
orange-red droplets that represent: 26. What is the significance of an APT test that remains pink
A. Fatty acids and soaps after addition of sodium hydroxide?
B. Fatty acids and neutral fats A. Fecal fat is present.
C. Fatty acids, soaps, and neutral fats B. Fetal hemoglobin is present.
D. Soaps C. Fecal trypsin is present.
D. Vitamin C is present.
19. When performing a microscopic stool examination for
muscle fibers, the structures that should be counted: 27. In the Van de Kamer method for quantitative fecal fat
A. Are coiled and stain blue determinations, fecal lipids are:
B. Contain no visible striations A. Converted to fatty acids prior to titrating with
sodium hydroxide
C. Have two-dimensional striations
B. Homogenized and titrated to a neutral endpoint with
D. Have vertical striations and stain red
sodium hydroxide
20. A value of 85% fat retention would indicate: C. Measured gravimetrically after washing
A. Dumping syndrome D. Measured by spectrophotometer after addition of
B. Osmotic diarrhea Sudan III
C. Secretory diarrhea 28. A patient whose stool exhibits increased fats, undigested
D. Steatorrhea muscle fibers, and the inability to digest gelatin may
have:
21. Which of the following tests would not be indicative of
steatorrhea? A. Bacterial dysentery
A. Fecal elastase-I B. A duodenal ulcer
B. Fecal occult blood C. Cystic fibrosis
C. Sudan III D. Lactose intolerance
D. Van de Kamer
22. The term “occult” blood describes blood that:
A. Is produced in the lower GI tract
B. Is produced in the upper GI tract
C. Is not visibly apparent in the stool specimen
D. Produces a black, tarry stool
3920_Ch14_255-268 23/01/14 10:39 AM Page 268

268 Part Three | Other Body Fluids

29. A stool specimen collected from an infant with diarrhea 30. Which of the following tests differentiates a malabsorp-
has a pH of 5.0. This result correlates with a: tion cause from a maldigestion cause in steatorrhea?
A. Positive APT test A. APT test
B. Negative trypsin test B. D-xylose test
C. Positive Clinitest C. Lactose tolerance test
D. Negative occult blood test D. Occult blood test

Case Studies and Clinical Situations


1. Microscopic screening of a stool from a patient exhibiting 3. A physician’s office laboratory is experiencing inconsisten-
prolonged diarrhea shows increased fecal neutrophils and cies in the results of patient-collected specimens for
normal qualitative fecal fats and meat fibers. FOBT. Patients are instructed to submit samples from two
a. What type of diarrhea do these results suggest? areas of three different stools. Positive and negative con-
trols are producing satisfactory results. Patient #1 is a 30-
b. Name an additional test that could provide more diag- year-old woman taking over-the-counter medications for
nostic information. gastric reflux who has reported passing frequent, black
c. Name one probable result for this test and one im- stools. The results of all three specimens are negative for
probable result. occult blood. Patient #2 is a 70-year-old woman suffering
d. If the test for fecal neutrophils were negative and the from arthritis. She is taking the test as part of a routine
fecal fat concentration increased, what type of diarrhea physical. The results of all three specimens are positive
for occult blood. Patient #3 is a 50-year-old man advised
would be suggested?
by the doctor to lose 30 lb. He has been doing well on a
2. Laboratory studies are being performed on a 5-year-old high-protein, low-carbohydrate diet. Two of his three
boy to determine whether there is a metabolic reason for specimens are positive for occult blood.
his continued failure to gain weight. In addition to having a. What is the possible nonpathologic cause of the unex-
blood drawn, the patient has a sweat chloride collected, pected results for Patient #1? Patient #2? Patient #3?
provides a random stool sample, and is asked to collect a b. How could the physician’s office staff avoid these dis-
72-hour stool sample. crepancies?
a. How can the presence of steatorrhea be screened for c. What testing methodology could be used for Patients #2
by testing the random stool sample? and #3?
b. How does this test distinguish among neutral fats, 4. A watery black stool from a neonate is received in the lab-
soaps, and fatty acids? oratory with requests for an APT test, fecal pH, and a
c. What confirmatory test should be performed? Clinitest.
d. Describe the appearance of the stool specimens if a. Can all three tests be performed on this specimen?
steatorrhea is present. Why?
e. If a diagnosis of cystic fibrosis is suspected, state two b. If the Clinitest is positive, what pH reading can be
screening tests that could be performed on a stool expected? Why?
specimen to aid in the diagnosis. c. The infant’s hemoglobin remains constant at 18 g/dL.
f. State a possible reason for a false-negative reaction in What was the significance of the black stool?
each of these tests. d. Would this infant be expected to have ketonuria? Why
g. What confirmatory test could be performed? or why not?
3920_Ch15_269-282 23/01/14 10:42 AM Page 281

Chapter 15 | Vaginal Secretions 281

Study Questions
1. Which of the following would not be a reason to collect a 8. A positive amine (Whiff) test is observed in which of the
vaginal fluid for analysis? following syndromes?
A. Vaginitis A. Bacterial vaginosis
B. Complications of pregnancy resulting in preterm B. Vulvovaginal candidiasis
delivery C. Atrophic vaginitis
C. Forensic testing in a sexual assault D. Desquamative inflammatory vaginitis
D. Pregnancy testing
9. A squamous epithelial cell covered with coccobacilli that
2. Which of the following organisms might not be detected extends beyond the cytoplasm margin is a:
if the specimen for vaginal secretion analysis had been A. Basal cell
refrigerated?
B. Parabasal cell
A. Prevotella bivia
C. Clue cell
B. Lactobacillus acidophilus
d. Blastospore
C. Trichomonas vaginalis
10. All of the following are diagnostic of bacterial vaginosis
D. Candida albicans
except:
3. The appearance of the vaginal discharge in vulvovaginal A. Vaginal pH of 3.8
candidiasis is described as:
B. Presence of clue cells
A. Clear and colorless
C. Positive amine (Whiff) test
B. Thin, homogeneous white-to-gray discharge
D. Thin, homogeneous white-to-gray vaginal
C. White, curd-like discharge
D. Yellow-green and frothy
11. Which of the following organisms produces lactic acid
4. A normal range for a vaginal pH is: and hydrogen peroxide to maintain an acid vaginal
A. 3.8 to 4.5 environment?
B. 5.0 to 6.0 A. Gardnerella vaginalis
C. 6.0 to 7.0 B. Mobiluncus spp.
D. 7.0 to 7.4 C. Lactobacilli spp.
D. β -Hemolytic streptococci
5. Which of the following tests differentiates budding yeast
cells from RBCs? 12. All of the following are diagnostic of vulvovaginal candidi-
A. pH asis except:
B. Saline wet mount A. Large numbers of WBCs
C. KOH prep B. Presence of clue cells
D. Whiff test C. Positive KOH test
D. Vaginal pH of 4.0
6. Which of the following constituents is normal in healthy
vaginal fluid secretions? 13. All of the following are diagnostic of trichomoniasis
A. Lactobacilli except:
B. Basal cells A. Vaginal pH of 6.0
C. Trichomonas vaginalis B. Positive amine test
D. Pseudohyphae C. Positive KOH test
D. Motile trichomonads present
7. Vaginal specimens collected for a saline wet prep should be:
A. Refrigerated to preserve motility
B. Prepared as soon as possible
C. Mailed to a reference laboratory
D. Preserved with potassium hydroxide
3920_Ch15_269-282 23/01/14 10:42 AM Page 282

282 Part Three | Other Body Fluids

14. The bacteria associated with desquamative inflammatory 15. The protein present in vaginal secretions that can identify
vaginitis is: patients who are at risk for preterm delivery is:
A. β -Hemolytic streptococci A. Human chorionic gonadotropin
B. Trichomonas vaginalis B. Estrogen
C. Gardnerella vaginalis C. PAMG-1
D. Mycoplasma hominis D. Fetal fibronectin

Case Studies and Clinical Situations


1. A 30-year-old woman has symptoms of dysuria, vaginal 3. During a routine visit with the gynecologist, a 60-year-old
itching, and a white, curd-like discharge. During her woman complained of vaginal dryness and soreness. During
visit at the Women’s Clinic, the patient revealed that she the examination, the health-care provider noted erythema
had recently completed a regimen of broad-spectrum of the vaginal mucosa. The pH of the vaginal secretions
antibiotics as treatment for a urinary tract infection. Her was 6.0. The KOH and amine (Whiff) tests were negative.
health-care provider takes a swab of the vaginal secretions The microscopic examination revealed epithelial cells, basal
for analysis. cells, decreased lactobacilli, and increased gram-positive
cocci and gram-negative rods.
a. What tests will be performed on the vaginal specimen?
a. What is the name of this condition?
b. Based on the patient history and observation of the
b. Explain how this condition can occur.
vaginal secretion, which test will be diagnostic for the
probable diagnosis? c. What is the treatment for this condition?
c. What confirmatory test can be performed?
d. What is the probable diagnosis?
e. What is the first choice of treatment?
2. A sexually active teenager visited the Women’s Clinic com-
plaining of vaginal itching and soreness. She indicated that
she was experiencing increased vaginal secretions that were
frothy and yellow to green. Upon examination, the health-
care provider noted a strawberry-like cervix and performed
a pH test on the secretions. The pH was 5.5 and the wet
prep demonstrated “swimming” organisms.
a. What is the probable diagnosis?
b. What other tests can be performed to confirm this
diagnosis?
c. What is the best course of treatment?
d. Should her sexual partner be treated?
e. List three complications that can occur with this
disorder.
3920_Ans_297-304 23/01/14 9:15 AM Page 297

Answers to Study Questions and Case


Studies and Clinical Situations

Chapter 1 d. Refer to the procedure manual. Check expiration


dates of controls and reagents. Open and test a new
Study Questions bottle of control or reagents.
e. When the controls are within range.
1. C 13. B 25. D
4. a. Delay in testing the specimen.
2. A 14. D 26. D
b. Incident report.
3. D 15. A 27. D
c. Delta check.
4. A 16. B 28. B
d. Treatment of the patient will be delayed because the
5. B 17. C 29. 2, 1, 2, 3, 2, 1 specimen will need to be recollected and tested. Extra
6. A 18. C 30. D expense incurred.
7. C 19. A 31. C e. Errors should be corrected as soon as possible follow-
8. D 20. B 32. D ing the institution’s policy. The original result must
9. D 21. D 33. C not be erased.
10. C 22. B 34. D
11. B 23. A Chapter 2
12. A 24. C
Study Questions
Case Studies and Clinical Situations 1. C 8. D 15. C
1. a. Review of the procedure by a designated authority 2. B 9. A 16. C
has not been documented. 3. C 10. A 17. A
b. Instructions and training are not being provided to 4. A 11. B 18. D
personnel performing collections. 5. D 12. B 19. B
c. A safety statement about the heat produced by the 6. C 13. D 20. B
reaction is not in the procedure manual.
7. C 14. A
d. The bottles have not been dated and initialled.
2. a. Correct; proficiency survey tests should be rotated Case Studies and Clinical Situations
among personnel performing the tests.
b. Accept; QC on the Clinitest tablets must only be 1. a. It could have increased pH, nitrite and bacteria
performed when they are used to perform a test. and decreased clarity, glucose, ketones, bilirubin,
urobilinogen and WBCs, RBCs and casts.
c. Correct; documentation of technical competency
should be performed on all personnel working in the b. The specimen was refrigerated and was brought
section and educational qualifications assessed. immediately to the laboratory.
3. a. The procedure was being performed incorrectly. 2. a. It would be less clear.
The correct timing of the glucose reaction was not b. Additional epithelial cells and bacteria (making it not
being done. acceptable for a culture).
b. The technologist performing the test. QC ensures 3. a. The results would be falsely decreased.
that the reagents and instrument are working prop- b. The patient needs to collect another specimen.
erly and that the technologist is performing the test 4. a. Yes.
correctly.
b. The temperature would be lower than body temperature.
c. At the beginning of each shift, when a new bottle
of reagent is opened or as stated in the procedure c. The specimen tested was not from the defendant.
manual. d. An accurately filled out chain of custody form (COC).

297
3920_Ans_297-304 23/01/14 9:15 AM Page 298

298 Answers to Study Questions and Case Studies and Clinical Situations

Chapter 3 Chapter 4
Study Questions Study Questions
1. B 14. B 23. B 1. A 9. D 17. D
2. D 15. B 24. D 2. D 10. A 18. D
3. C 16. D 25. D 3. A 11. C 19. C
4. D 17. B 26. C 4. D 12. B 20. B
5. A 18. A. Beta2-mi- 27. C 5. A 13. D 21. B
6. B croglobulin; 28. B 6. A 14. A 22. A
B. Creatinine;
7. C 29. A 7. C 15. C 23. B
C. Cystatin
8. D C; D. 125I- 30. +0.5 8. C 16. B 24. D
9. B iodothalamate 31. D
10. A 19. B 32. 600 mL/min Case Studies and Clinical Situations
11. C 20. 69 mL/min 33. C 1. a. An elevated pH and a positive reagent strip reaction
12. D 21. D 34. B for nitrite.
13. D 22. D b. The reagent strip specific gravity would be much
lower if the patient had been given radiographic dye.
Case Studies and Clinical Situations c. The reagent strip test for bilirubin would be positive.
d. The reagent strip reaction for blood would be positive
1. a. 160-mg/dL to 180-mg/dL.
and red blood cells would be seen in the microscopic.
b. Renal tubular reabsorption is impaired.
2. a. 1.018
2. a. Juxtaglomerular apparatus → Angiotensinogen
b. Yes.
→ Renin → Angiotensin I → Angiotensin II.
c. It would agree with the reagent strip reading because
b. Vasoconstriction, increased sodium reabsorption, and
like the osmometer, the reagent strip is not affected
increased aldosterone to retain sodium.
by high-molecular-weight substances.
c. Production of renin decreases and, therefore, the
3. Hemoglobin and myoglobin.
actions of the renin-angiotensin-aldosterone system.
a. Examine the patient’s plasma /serum. The breakdown
3. a. The physician can calculate the approximate creatinine
of red blood cells to hemoglobin produces a red
clearance using the MDRD-IDMS-traceable formula.
serum. Myoglobin is produced from skeletal muscle
b. The cystatin C test and the beta2-microglobulin test and is rapidly cleared from the plasma/serum.
serum tests.
4. a. Mrs. Smith has been eating fresh beets.
c. No. The beta2-microglobulin test requires a normal
b. Yes. The pH of Mrs. Smith’s urine is acidic or she has
immune system and malignancies can affect the
not recently consumed fresh beets.
immune system; therefore, the test cannot be reli-
able in patients with immunologic disorders and 5. No. The urine can contain increased pH, glucose, ketones,
malignancies. bilirubin, urobilinogen, nitrite, and small amounts of
cellular structures.
4. a. Yes. Serum from the midnight specimen is not being
separated from the clot and refrigerated in a timely
manner.
Chapter 5
b. Lactic acid will be present in serum that is not
separated from the clot and will affect the freezing Study Questions
point osmolarity readings.
1. A 8. B 15. B
c. If the laboratory is using a freezing point osmometer,
results will be affected by alcohol ingestion; vapor 2. D 9. D 16. A
pressure results would not be affected and could be 3. A 10. 2,1,2,3,1,2,3 17. C
used as a comparison. 4. C 11. B 18. A
5. a. Diabetes insipidus. 5. D 12. A 19. A
b. Neurogenic diabetes insipidus. 6. A 13. A 20. C
c. Nephrogenic diabetes insipidus. 7. D 14. D 21. A
3920_Ans_297-304 23/01/14 9:15 AM Page 299

Answers to Study Questions and Case Studies and Clinical Situations 299

22. B 32. B 42. B 6. a. No, the specimen is clear.


23. C 33. A 43. D b. Myoglobinuria.
24. A 34. 1,3,4,2 44. C c. Muscle damage from the accident (rhabdomyolysis).
25. C 35. A 45. B d. Yes. Myoglobin is toxic to the renal tubules.
26. B 36. D 46. C 7. a. Laboratory personnel are not tightly capping the
27. A 37. C 47. C reagent strip containers in a timely manner.
28. D 38. A 48. A b. Personnel performing the CLIA-waived reagent strip
test are not waiting 2 minutes to read the LE reaction.
29. A 39. C 49. C
c. The student is not mixing the specimen.
30. C 40. A
d. The reagent strips have deterioated and the quality
31. 1, 2, 1, 2, 1, 2 41. B
control on the strips was not performed prior to
reporting the results.
Case Studies and Clinical Situations
1. a. The blood glucose is elevated and has exceeded the
renal tubular maximum (Tm) for glucose. Chapter 6
b. Diabetes mellitus.
Study Questions
c. It indicates diabetes mellitus related renal disease.
1. A 18. D 35. A
d. Renal tubular reabsorption disorders.
2. D 19. B 36. C
2. a. Yellow foam.
3. C 20. C 37. A
b. Possible biliary-duct obstruction preventing bilirubin
from entering the intestine. 4. C 21. A 38. D
c. Icteric. 5. A 22. B 39. A
d. Protection from light. 6. B 23. C 40. C
3. a. Hemoglobinuria. 7. C 24. D 41. D
b. Increased hemoglobin presented to the liver results 8. D 25. D 42. A
in increased bilirubin entering the intestine for 9. C 26. B 43. A
conversion to urobilinogen. 10. D 27. D 44. C
c. The circulating bilirubin is unconjugated. 11. D 28. A 45. D
d. It would if a multisix reagent strip is used and would 12. A 29. B 46. C
not if a Chemstrip is used. A Watson-Schwartz test is
13. C 30. C 47. 4, 3, 5, 1
more specific for porphobilinogen.
14. B 31. C 48. 3, 5, 2, 6, 4
4. a. Negative chemical reactions for blood and nitrite.
Ascorbic acid interference for both reactions. A ran- 15. C 32. D 49. 4, 8, 7, 6, 1, 5
dom specimen or further reduction of nitrite could 16. A 33. D 50. 3, 5, 2, 1, 7, 4
cause the negative nitrite. 17. D 34. B
b. Glucose, bilirubin, LE. Ascorbic acid is a strong re-
ducing agent that interfers with the oxidation reaction Case Studies and Clinical Situations
in the glucose test. Ascorbic acid combines with the
diazo reagent in the bilirubin and LE tests, lowering 1. a. Yeast grows best at a low pH with an increased con-
the sensitivity. centration of glucose.
c. The dark yellow color may be caused by beta-carotene b. Yes, this exceeds the renal threshold.
and vitamin A, and some B vitamins also produce c. No, yeast is not capable of reducing nitrate to
yellow urine. nitrite.
d. Non-nitrite–reducing microorganisms; lack of dietary d. Moderate blood with no RBCs.
nitrate; antibiotic administration. e. Myoglobin is the cause of the positive chemical test
5. a. To check for possible exercise-induced abnormal results. result for blood. The patient has been bed-ridden
b. Negative protein and blood, possible changes in color for an extended period of time, causing muscle
and specific gravity. destruction.
c. Renal.
3920_Ans_297-304 23/01/14 9:15 AM Page 300

300 Answers to Study Questions and Case Studies and Clinical Situations

2. a. The large objects are in a different plane from that of g. No, calcium carbonate crystals are found in alkaline
the urinary constituents. urine; therefore, clumps of amorphous phosphates
b. Contamination by artifacts. may be present.
c. No, because they are in a different plane.
d. Polarizing microscopy. Chapter 7
3. a. Renal tubules.
b. Yes, viral infections can cause tubular damage. Study Questions
c. RTE cells absorb the bilirubin-containing urinary
filtrate. 1. B 8. D 15. A
d. Liver damage inhibits processing of reabsorbed 2. C 9. A 16. C
urobilinogen. 3. B 10. C 17. A
e. Hemolytic anemia. 4. C 11. C 18. A
4. a. The patient is taking a pigmented medication, such as 5. B 12. C 19. A
phenazopyridine. 6. D 13. B 20. D
b. Yes. 7. C 14. D
c. Ask what medications the patient is taking.
d. Ampicillin. Case Studies and Clinical Situations
5. a. Calcium oxalate. 1. a. Acute glomerulonephritis.
b. Monohydrate and dihydrate calcium oxalate. b. M protein in the cell wall of the group A
c. Oval: monohydrate; envelope: dihydrate. streptococcus.
d. Monohydrate. c. Glomerular bleeding.
6. a. Microscopic results do not match the chemical tests d. No, they are also passing through the damaged
for blood, nitrite, and leukocyte esterase. glomerulus.
b. The specimen has been unpreserved at room temper- e. Good prognosis with appropriate management of
ature for too long, the cells have disintegrated, and secondary complications.
the bacteria have converted the nitrite to nitrogen. f. Henoch-Schönlein purpura.
c. The pH. 2. a. IgA nephropathy.
d. Ask the clinic personnel to instruct the patient to col- b. Serum IgA level.
lect a midstream clean-catch specimen and have the
c. Chronic glomerulonephritis/end-stage renal
specimen delivered immediately to the laboratory.
disease.
7. a. No, because they are associated with strenuous
d. Impaired renal tubular reabsorption associated with
exercise.
end-stage renal disease.
b. The positive blood reaction is from hemoglobinuria
e. The specific gravity is the same as that of the ultrafil-
or myoglobinuria resulting from participating in a
trate, indicating a lack of tubular concentration.
contact sport. The protein is orthostatic.
f. The presence of extreme urinary stasis.
c. Increased excretion of RTE cell lysosomes in the
presence of dehydration. 3. a. Nephrotic syndrome.
8. a. Yes, the waxy casts are probably an artifact such as b. Nephrotic syndrome may be caused by sudden,
a diaper fiber. Waxy casts are not associated with severe hypotension.
negative urine protein. c. Changes in the electrical charges of the shield of
b. No, this is normal following an invasive procedure. negativity produce increased membrane permeability.
c. Yes, tyrosine crystals are seen in severe liver disease; d. Decreased plasma albumin lowers the capillary on-
therefore, the bilirubin should be positive. The cotic pressure, causing fluid to enter the interstitial
crystals may be an artifact or from a medication. tissue.
d. Yes, uric acid crystals may be mistaken for cystine e. Reabsorption of filtered lipids by the RTE cells.
crystals. 4. a. Minimal change disease.
e. Yes, radiographic dye crystals associated with a high b. Nephrotic syndrome, focal segmental
specific gravity resemble cholesterol crystals. glomerulosclerosis.
f. No, Trichomonas is carried asymptomatically by men. c. Good prognosis with complete remission.
3920_Ans_297-304 23/01/14 9:15 AM Page 301

Answers to Study Questions and Case Studies and Clinical Situations 301

5. a. Goodpasture syndrome. Chapter 8


b. The autoantibody attaches to the glomerular capillar-
ies, causing complement activation and destruction of Study Questions
the capillaries.
1. A 10. D 19. B
c. Wegener’s granulomatosis.
2. C 11. D 20. B
d. Antineutrophilic cytoplasmic antibody.
3. B 12. B 21. D
e. Granuloma formation resulting from autoantibodies
binding to neutrophils in the vascular walls and 4. C 13. C 22. D
initiating an immune response. 5. A 14. A 23. D
6. a. Cystitis, UTI. 6. A 15. D 24. C
b. The specimen is very dilute. 7. C 16. B, A, B, B, A 25. B
c. Irritation of the urinary tract will cause a small 8. B 17. D 26. D
amount of bleeding. The cells and bacteria may cause 9. D 18. B 27. D, F, A, E, C, B
a trace protein or it may be a false-positive due to the
high pH. Case Studies and Clinical Situations
d. Yes, glitter cells are seen in hypotonic urine.
1. a. Underdevelopment of the liver.
e. Female children.
b. Yes, with severe acquired liver disease.
f. Pyelonephritis.
c. Tyrosine crystals; leucine crystals, bilirubin crystals.
7. a. Intravenous pyelogram.
d. Protect the specimen from light.
b. Chronic pyelonephritis.
2. a. Isovaleric acidemia.
c. WBC cast.
b. Maple syrup urine disease.
d. Reflux nephropathy.
c. Yes, the MS/MS screen would be positive.
e. Performing a Gram stain.
3. a. Renal lithiasis.
f. Radiographic dye.
b. Impaired renal tubular reabsorption of cystine.
g. Permanent tubular damage and progression to
chronic, end-stage renal disease. c. Lysine, arginine, ornithine.
8. a. Abnormal. d. They are more soluble than is cystine.
b. Acute interstitial nephritis. e. The disorder is inherited.
c. This disorder is an inflammation not an infection. 4. a. Yes.
d. Discontinue the medication because it is causing the b. Yes, uric acid crystals accumulating on the surface of
allergic reaction. the diaper could have an orange color.
9. a. Acute renal failure. c. Lesch-Nyhan disease.
b. The prerenal sudden decrease in blood flow to the d. Yes, the disease is inherited as a sex-linked recessive.
kidneys. e. Hypoxanthine guanine phosphoribosyltransferase.
c. Lack of renal concentrating ability. 5. a. Yes. The urine may contain melanin or homogentisic
d. Tubular damage. acid.
e. The increased diameter of the damaged distal convo- b. Yes. Melanin will react with sodium nitroprusside, the
luted tubule and extreme urinary stasis allowing casts reagent used on reagent strips for the detection of ketones.
to form in the collecting ducts. c. Yes. Homogentisic acid turns black in alkaline urine.
10. a. Renal lithiasis. 6. a. Yes, the purple blue color could indicate the presence
b. The high specific gravity. of indican in the urine.
c. Yes, the dark yellow color and high specific gravity b. Hartnup disease.
indicate a concentrated urine, which induces the c. Good with proper dietary supplements.
formation of renal calculi. 7. a. The Ehrlich reaction
d. Calcium oxalate. b. Acetylacetone.
e. Increased hydration and dietary changes. c. Porphobilinogen.
d. Blood.
e. Free erythocyte protoporphyrin (FEP).
3920_Ans_297-304 23/01/14 9:15 AM Page 302

302 Answers to Study Questions and Case Studies and Clinical Situations

Chapter 9 5. a. Stain precipitate is being confused with Gram-


positive cocci.
Study Questions b. Differentials are being reported from the counting
chamber.
1. B 13. A 25. C
c. The albumin is contaminated.
2. C 14. B 26. A
d. The specimens are not being promptly delivered to
3. B 15. C 27. C the laboratory.
4. A 16. D 28. A
5. B 17. D 29. C
6. B, B, A, A 18. D 30. D
Chapter 10
7. C 19. B 31. B Study Questions
8. A 20. D 32. A
1. C 12. C 23. D
9. C 21. A 33. A
2. D 13. A 24. B
10. C 22. B 34. C
3. B 14. A 25. D
11. D 23. B 35. C
4. C 15. B 26. B
12. D 24. B 36. D
5. D 16. B 27. D
6. A 17. A 28. B
Case Studies and Clinical Situations
7. B 18. C 29. C
1. a. Cerebral hemorrhage because of the presence of
8. D 19. A 30. A
erythrophagocytosis, even distribution of blood, and
patient’s history. 9. C 20. C 31. B
b. No, they would be consistent with peripheral blood 10. B 21. A
entering the CSF. 11. B 22. A
c. No, they are consistent with the percentages seen in
peripheral blood. Case Studies and Clinical Situations
d. Hemosidern granules and hemotoidin crystals. 1. a. Sperm concentration, motility, and morphology.
e. A traumatic tap. b. 21,000,000; no.
2. a. An India ink preparation. c. 1,800,000; no.
b. Cryptococcus meningitis. d. Yes. The normal sperm concentration is 20 to
c. Immunologic testing for Cryptococcus. 60 million/mL. Spermatid counts over 1 million
d. Rheumatoid factor. are considered abnormal. Both of these abnormal
results and the abnormal motility are related to de-
e. Acid-fast staining and culture.
fects in sperm maturation.
f. Noticeable oligoclonal bands in both the CSF and
2. a. Male antisperm antibodies may form following
serum.
vasovasostomy procedures.
3. a. CSF/serum albumin index = 6.7.
b. The MAR test and the immunobead test.
b. Yes.
c. The MAR test detects the presence of IgG male sperm
c. IgG index = 1.5. antibodies. The immunobead test delineates the areas
d. Immunoglobulin synthesis within the CNS. of the sperm (head, tail, neck) that are affected by the
e. Multiple sclerosis. antibodies.
f. Oligoclonal banding only in the CSF. d. Clumping, ovum penetration, and motility.
g. Myelin basic protein. 3. The specimen contains urine, which is toxic to sperm,
therefore decreasing viability.
4. a. Viral, tubercular, or fungal meningitis.
4. The specimen was improperly collected, and the first
b. No, the Gram stain would be negative in viral and
part of the ejaculation was lost.
tubercular and not always positive in fungal meningitis.
5. a. Yes, there is insufficient prostatic fluid present.
c. Yes. Lymphocytes are very predominant in viral
meningitis. b. Zinc, citrate, and acid phosphatase.
d. Yes, a CSF lactate level of 25 mg/dL or less would aid c. Sperm motility is severely affected.
in confirming bacterial meningitis. The lactate level 6. a. Acid phosphatase and seminal glycoprotein p30 tests.
would be higher in tubercular and fungal meningitis. b. Microscopic examination for the presence of sperm.
3920_Ans_297-304 23/01/14 9:16 AM Page 303

Answers to Study Questions and Case Studies and Clinical Situations 303

Chapter 11 Case Studies and Clinical Situations


1. a. Pleural fluid.
Study Questions
b. Transudate, because all the test results are consistent
1. B 9. B 18. D with those of a transudate.
2. A 10. C 19. B c. Pleural fluid to serum ratios of cholesterol and
3. A 11. A 20. Negative bilirubin.
4. B 12. A 21. C 2. a. Pneumonia.
5. B, C, B, A, D, 13. C 22. A b. Chest tube drainage.
B, D 14. A 23. C 3. a. 1.6.
6. A 15. B 25. A b. Transudate. The SAAG is above 1.1.
7. B 16. B c. Hepatic disorder.
8. B 17. A 4. a. To differentiate between cirrhosis and peritonitis;
cirrhosis.
Case Studies and Clinical Situations b. Pancreatitis or gastrointestinal perforation; alkaline
phosphatase.
1. a. 1. Sterile, heparinized tube, liquid EDTA tube,
c. Rupture or accidental puncture of the bladder.
nonanticoagulated tube.
d. To detect the presence of gastrointestinal (CEA) and
b. MSU crystals are seen in gout.
ovarian (CA 125) cancers.
c. Highly birefringent, needle-shaped crystals under
5. The patient has been a victim of blunt trauma and the
polarized light that turn yellow when aligned with
physician wants to determine if abdominal bleeding is
the slow vibration of red compensated polarized light.
occurring; abdominal bleeding.
d. Infection is frequently a complication of severe
6. Thyroid profile; CA 125.
inflammation.
2. a. WBC diluting fluid containing acetic acid was used.
b. Normal, hypotonic, or saponin-containing saline Chapter 13
should be used.
c. Crystal-induced inflammatory and septic. Study Questions
d. Gram stain and culture, crystal examination. 1. B 8. 2, 4, 1, 3 15. C
3. a. Noninflammatory. 2. C 9. A 16. True
b. Hydroxyapatite crystals. 3. A 10. C 17. C
c. Glucose. A normal result is consistent with nonin- 4. C 11. B 18. B
flammatory arthritis. 5. B 12. True 19. B
4. a. Fibrinogen. 6. A 13. A 20. D
b. EDTA or heparinized tube. 7. D 14. True
c. No, the bacteria will be trapped in the clot.
Case Studies and Clinical Situations
Chapter 12 1. a. Yes.
b. FLM.
Study Questions c. The level of phosphatidylglycerol present in the fetal
1. C 9. B 18. B lungs.
2. D 10. C 19. C d. Phosphatidylglycerol is essential for FLM, and levels
3. A 11. D 20. B do not always parallel lecithin levels in fetuses of
diabetic mothers.
4. D 12. D 21. B
2. a. A neural tube disorder such as spina bifida or
5. C 13. D 22. B anencephaly.
6. D 14. C 23. C b. An acetylcholinesterase level.
7. B, A, A, A, B, 15. B 24. B c. The amniotic fluid specimen contains blood.
A, B 16. A 25. D
8. B 17. D 26. D
3920_Ans_297-304 23/01/14 9:16 AM Page 304

304 Answers to Study Questions and Case Studies and Clinical Situations

3. a. False-positive result. 3. a. Patient #1: gastric reflux medication containing


b. False-positive result. bismuth may produce black stools; Patient #2:
medications such as aspirin and other NSAIDs may
c. No effect.
cause gastric bleeding; Patient #3: red meat was not
d. False-positive result. avoided for 3 days prior to sample collection.
4. a. False-positive result. b. Provide dietary and medication instructions to patients.
b. False-positive result. c. The Hemoccult ICT immunochemical test.
c. False-positive or test interference. 4. a. The APT test cannot be performed because the
d. No effect. hemoglobin is already denatured.
5. a. The specimen was exposed to light, possible wrong b. The pH will be low because increased carbohydrates
specimen sent. are available for bacterial metabolism.
c. The infant had ingested maternal blood.

Chapter 14 d. Yes, adequate carbohydrates are not present, and fats


are being metabolized for energy.
Study Questions
1. C 11. A 21. B Chapter 15
2. A 12. C 22. C
3. C 13. D 23. B
Study Questions
4. B 14. B 24. A 1. D 6. A 11. C
5. D 15. B 25. B 2. B 7. B 12. B
6. D 16. False 26. B 3. C 8. A 13. C
7. D 17. C 27. A 4. A 9. C 14. A
8. C 18. C 28. C 5. C 10. A 15. D
9. D 19. C 29. C
10. C 20. D 30. B
Case Studies and Clinical Situations
1. a. Vaginal pH, saline and KOH wet preps, Gram stain.
Case Studies and Clinical Situations b. KOH will reveal budding yeast.
1. a. Secretory diarrhea. c. Culture and DNA direct hybridization probe
(Affirm VPIII).
b. Stool culture.
d. Vulvovaginal candidiasis caused by Candida albicans.
c. Probable: Salmonella, Shigella, Campylobacter, Yersinia,
E. coli; Improbable: Staphylococcus, Vibrio. e. Antifungal agents.
d. Osmotic diarrhea. 2. a. Trichomoniasis caused by Trichomonas vaginalis.
2. a. Microscopic examination for fecal fats. b. Wet mount, vaginal pH, amine test from KOH prep,
DNA probe (Affirm VPIII), OSOM Trichomonas
b. Neutral fats stain directly and appear as large, orange-
Rapid Test.
red droplets; soaps and fatty acids appear as smaller
orange-red droplets after pretreatment of the specimen c. Metronidazole.
with heat and acetic acid. d. Yes.
c. Quantitative fecal fat test. e. Complications include low birth rate, premature rupture
d. Bulky and frothy. of membranes, preterm delivery during pregnancy.
e. Muscle fiber screening and the gelatin test for trypsin. 3. a. Desquamative inflammatory vaginitis secondary to
atrophic vaginitis.
f. Muscle fiber: failure to include red meat in the diet;
gelatin test: intestinal degradation of trypsin or the b. Reduced estrogen production in postmenopausal
presence of trypsin inhibitors. women.
g. Chymotrypsin or elastase I. c. Hormone replacement therapy (estrogen).
3920_Abbreviations_305-306 23/01/14 9:14 AM Page 305

Abbreviations

AABB American Association of Blood Banks DNPH 2,4-dinitrophenylhydrazine


A:C albumin:creatinine [ratio] EDS early dumping syndrome
ACE angiotensin-converting enzyme EDTA ethylenediaminetetraacetic acid
AChE acetylcholinesterase eGFR estimated glomerular filtration rate
ADA adenosine deaminase ELISA enzyme-linked immunoabsorbent assay
ADH antidiuretic hormone EPS expressed prostatic secretions
AFP alpha-fetoprotein EQA external quality assessment
AGN acute glomerulonephritis EQC external quality control
AHG antihuman globulin EU Ehrlich unit
AIDS acquired immunodeficiency syndrome FAH fumarylacetoacetate hydrolase
AIN acute interstitial nephritis FDA Food and Drug Administration
ALA ␣ -aminolevulinic acid FEP free erythrocyte protoporphyrin
ANA antinuclear antibody fFN fetal fibronectin
ANCA antineutrophilic cytoplasmic antibody FISH fluorescence in situ hybridization
AOA American Osteopathic Association FLM fetal lung maturity
ARF acute renal failure FOBT fecal occult blood testing
ART assisted reproductive technology FSGS focal segmental glomerulosclerosis
ASHI American Society of Histocompatibility and FTA-ABS fluorescent treponemal antibody-absorption
Immunogenetics GALT galactose-1-phosphate uridyl transferase
ATN acute tubular necrosis gFOBT guaiac fecal occult blood test
BAL bronchoalveolar lavage GFR glomerular filtration rate
BAT bacterial antigen test GI gastrointestinal
BSI body substance isolation GTT glucose tolerance test
BUN blood urea nitrogen 1-H NMR hydrogen nuclear magnetic resonance
BV bacterial vaginosis spectroscopy
CAP College of American Pathologists H+ titratable acid/hydrogen ion
CASA computer-assisted semen analysis H2PO4 hydrogen phosphate ion
CDC Centers for Disease Control and Prevention Hb hemoglobin
CEA carcinoembryonic antigen HBV hepatitis B virus
CHP chemical hygiene plan HCV hepatitis C virus
CLIA Clinical Laboratory Improvement Amendments HCO3 bicarbonate ion
CLRW clinical laboratory reagent water HDN hemolytic disease of the newborn
CLSI Clinical and Laboratory Standards Institute HICPAC Healthcare Infection Control Practices
CNS central nervous system Advisory Committee
COC chain of custody 5-HIAA 5-hydroxyindoleacetic acid
COLA Commission on Laboratory Assessment HIV human immunodeficiency virus
CPPD calcium pyrophosphate dihydrate HLA-B12 human leukocyte antigen-B12
CPR cardiopulmonary resuscitation hpf high-power field
CSF cerebrospinal fluid HRCT high-resolution computerized tomography
CTAB cetyltrimethylammonium bromide HSV herpes simplex virus
Cu2O cuprous oxide IBS irritable bowel syndrome
CuSO4 copper sulfate IDMS isotope dilution mass spectrophotometry
CV coefficient of variation IEF isoelectric focusing
DBDH diisopropyl benzene dihydroperoxide IEM inborn error of metabolism
DCT distal convoluted tubule IFE immunofixation electrophoresis
DI diabetes insipidus iFOBT immunochemical fecal occult blood test
DIDNTB (3⬘,3⬙, diodo 4⬘,4⬙-dihydroxy-5⬙,5⬙- IgA immunoglobulin A
dinitrophenyl)-3,4,5,6-tetra-bromo- IgG immunoglobulin G
sulphonphtalein IgM immunoglobulin M
DIV desquamative inflammatory vaginitis ISO International Organization for Standardization

305
3920_Abbreviations_305-306 23/01/14 9:14 AM Page 306

306 Abbreviations

IUI intra-uterine insemination PEP postexposure prophylaxis


IVF in vitro fertilization PG phosphatidyl glycerol
JC Joint Commission PKU phenylketonuria
KOH potassium hydroxide PM preventive maintenance
LAF lateral flow assay POC point of care
LBC lamellar body count PPE personal protective equipment
LD lactate dehydrogenase PPMT pre- and post-massage test
LE leukocyte esterase PSA prostate-specific antigen
LED light-emitting diode PSP phenolsulfonphthalein
LIS laboratory information system PT proficiency testing
lpf low-power field QA quality assurance
L/S lecithin-sphingomyelin ratio QC quality control
MAR mixed agglutination reaction RA rheumatoid arthritis
MBP myelin basic protein RAAS renin-angiotensin-aldosterone system
MDRD modification of diet in renal disease RBC red blood cell
MESNA mercaptoethane sulfonate sodium RCF relative centrifugal force
MoM multiples of the median RDS respiratory distress syndrome
mOsm milliosmole RF rheumatoid factor
MPGN membranoproliferative glomerulonephritis RGE rapid gastric emptying
MSAFP maternal serum alpha-fetoprotein RPM revolutions per minute
MS/MS tandem mass spectrophotometry RPR rapid plasma reagin
MSDS Material Safety Data Sheet RTE renal tubular epithelial (cells)
MSU monosodium urate (uric acid) SAAG serum-ascites albumin gradient
MSUD maple syrup urine disease SD standard deviation
NaCl sodium chloride SKY fluorescent mapping spectral karyotyping
NFPA National Fire Protection Association SLE systemic lupus erythematosus
NH4+ ammonium ion SSA sulfosalicylic acid
NIRS near-infrared spectroscopy STI sexually transmitted infection
NKDEP National Kidney Disease Education Program TAT turnaround time
NRBC nucleated red blood cell Tm maximal reabsorptive capacity/tubular
NTD neural tube defect reabsorptive maximum
OD optical density TmG maximal tubular reabsorption capacity for
OSHA Occupational Safety and Health glucose
Administration TMB 3,3⬘,5,5⬘-tetramethylbenzidine
PAH p-aminohippuric acid UP Universal Precautions
PAMG-1 placental ␣ 1-microglobulin UTI urinary tract infection
PB peripheral blood VDRL Venereal Disease Research Laboratory
PCR polymerase chain reaction WBC white blood cell
PCT proximal convoluted tubule WHO World Health Organization

You might also like